Download as pdf or txt
Download as pdf or txt
You are on page 1of 90

1

By Medical Students

For Medical Students

2014

This is not intended to be your reviewer. Your best reviewer is of Barbara Bates,
your best handout is your ear, and your best teacher is yourself



CLINICAL NEUROLOGY by Dra Rosales







3

HEAD, EYES, EARS, NOSE, THROAT by Dra Solis






8

EXTREMITIES by Dr Paulino









34

CHEST and LUNGS, BREAST by Dra Lee







56

CARDIOLOGY by Dra Deduyo









65

ABDOMEN by Dra Cortez









76







Please be guided that answers are coming from medical students; do not rely.

AKO NA MISMO NAGSASABING MALI MALI UNG IBANG MGA SAGOT DITO KASI AYAW MAKINIG NG IBA NA NAG EENCODE
SABING WAG PALITAN UNG SAGOT.


- N





























NEUROLOGY
1. Gait ataxia and hypotonia will be seen in a patient with a lesion in the
a. Rostral vermis b.Posterior lobe c.Caudal vermis
d.All e. A and C
2. Manifestation of vermian lesion
a. Dysarthria
b. Scanning speech
c. Nystagmus
d.All
e.A and c
3. Manifestation of cerebellar dysfunction
a. Hypotonia
b. Decomposition of movement c. Mild aesthenia d.All
e. A and C
4. Test for arm dystaxia
a. Wrist tapping test
b. Arm pulling test
c. Thigh patting test
d.All e.A and C
5. True in a patient with cerebellar hemisphere infarct in the right
a. Nystagmus
b. Limb ataxia C. Dysmetria L d. All e. A and B
6. Superficial sensation routinely examined
a. Romberg
b. Asteriognosis c. Joint position
d. Pain e. C and D
7. Manifestation of polyneuropathy
a. Symetrical distal weakness
b. Areflexia
c. Preferential sensory loss in proximal limb

d. A and B

e. All
8. From medial to lateral (CTLS) segmented arrangement of fibers in the spinal cord is seen in the
a. Fascicular cuneatus/ gracilis
b. Lateral cortico spinal c. Spinothalamic
d. All e. B and C
9. TRUE during testing of sensory function,
a. Usually done with patients eyes closed
b. Should compare cornified vs. Non cornified areas
c. Should examine symmetrical dermatomal area
d. All
e. A and C
10. Pins and needles sensation
a. Dysaesthesia
b. Paresthesia c. Alodynia
e. Analgesia
11. Biceps reflex is subserved
a. C2 c3
b.C4-c5
c. C5-c6 d. C7-c8
12. Variations of babinski
a. Snout reflex
c. Hoffmans
d. Chaddocks e.All
b. B and C
13. Examination of motor function
a. Look for involuntary movements
c. Inspect muscle size
b. Look for coordinated performance of
d. all
motor acts
14. TRUE of spastic gate:
a. Narrow base of support
b. Foot plantar flexed and everted
c. Leg externally rotated at the hip
d. All
e. A and C
15. Upper motor neuron paralysis:
a. Muscles affected in groups
e. A and C
b. Rigidity
c. Babinski
d. All

4
16.
17.
18.
19.
20.

Fundoscopy
D
Nystagmus
C
Say EGG A

Open mouth
B
Corneal blink reflex


a. Cranial nerve from medulla

b. Cranial nerve from the pons
c. Cranial nerve from the midbrain

d. Cranial nerve from the supra tentorial

A. MATCHING TYPE:
1. A stoke patient can open his eyes, moans and flexes on painful stimulation has a Glasgow coma scale of:
A. 3/15
B. 6/15
C. 10/15
D. 12/15
2. The main objective of doing deep tendon reflex is to:
A. Differentiate whether the lesion is UMN or LMN
B. Differentiate whether the lesion is brain stem or spinal cord
C. Differentiate whether the lesion is anterior horn cell or peripheral nerve
D. All of the above
3. Babinski is not seen in:
A. Frontal lobe tumor
B. Brainstem stroke
C. Spinal cord
D. Diabetic
compression
neuropathy
4. A high stepped, slapping gate is usually secondary to:
A. Parkinsons disease
B. Posterior column
C. UMN Lesion
D. Gait apraxia
lesion
5. Paralysis of upward gaze is usually secondary to:
A. Optic nerve lesion
B. Optic chiasm lesion
C. Pineal lesion
D. Pituitary Lesion
6. Not a feature of metabolic encephalopathy
A. Pupils are equal and reactive
C. Common occurrence of movement abnormality
B. Severe mental status alteration
D. Presence of lateralizing sign
7. Fatigue with exercise is usually seen in:
A. Neuropathy
B. Myopathy
C. Neuromuscular function disorder D. UMN lesion
8. A unilateral, dilated, non reactive pupil in a comatose pate will indicate:
A. Metabolic encephalopathy
C. Pontine hemorrhage
B. Temporal bone herniation
D. Drug overdose
9. A patient complaining of weakness that he can only move his extremities against gravity is graded as:
A. 1/5
B. 2/5
C. 3/5
D. 4/5
10. Mental status examination is a test for the integrity of:
A. Cerebral cortex
B. Cerebellum
C. Brainstem
D. Cranial nerves
11. An optic chiasm lesion would cause:
A. Failure of
B. Paralysis of upward
C. Bitemporal Hemianopsia D. All of the above
convergence
gaze
12. Lesion in the cavernous sinus would involve the following cranial nerves, except:
A. CN III
B. CN IV
C. CN V
D. CN VI
13. Weakness of knee extension is caused by a lesion of:
A. Femoral nerve
B. Peroneal nerve
C. Sciatic Nerve
D. Popliteal nerve
14. Features of upper motor neuron, except:
A. Spastic
B. (+) Babinski
C. Atrophy
D. None of the above
15. Foot drop is secondary to a lesion in the:
A. Femoral nerve
B. Peroneal nerve
C. Lateral cutaneous
D.
nerve
16. The first cranial nerve to be affected by increased intracranial pressure is:
A. CN III
B. CN IV
C. CN VI
D. CN VI
17. Signs of peripheral nerve lesion, except:
A. Atrophy
B. Fasciculations
C. Spasticity
D. Hyporeflexia
18. Neurologic examination in a comatose patient includes the following, except:
A. Fundoscopy
B. Pupillary light reflex
C. Mental status
D. Cerebellar examination
examination
19. Signs and symptoms of myopathy includes the following, except:
A. Proximal weakness
B. Atrophy
C. Hyperreflexia
D. None of the above
20. The most reliable sign of UMN lesion:
A. Hyperreflexia
B. Atrophy and fasciculation C. (+) Babinski
D. Spasticity
21. Presence of cheery red spots seen in the retina by fundoscopy is seen in:
A. Tay Sachs disease
C. Inreacrania hemorrhage

B. Subarachnoid hemorrhage
D. Von Recklinghausens disease
22. Presence of caf au lait spots seen in fundoscopy is secondary to
A. Tay Sachs disease
C. Inreacrania hemorrhage
B. Subarachnoid hemorrhage
D. Von Recklinghausens disease
23. Ataxia is seen in the following lesions, except:
A. Dorsal column
B. Cerebellum
C. Spinocerebellar tract
D. None of the above
24. Movement disorder associated with basal ganglia lesions, except:
A. Dysmetria
B. dystonia
C. Bradykinesia
D. Tremor



25. In the lesion of the superior orbital fissure the following are true, except:
A. Weakness of the lateral movement of the eye
B. Weakness of the medial movement of the eye
C. Weakness of the superior and inferior movement of the eye
D. None of the above

B. ASSOCIATION TYPE
A If 1,2,3 are correct
D Only 4 is correct
B If 1 and 3 are correct
E if all are correct
C If 2 and 4 are correct

E 26. Neurologic examination is mandatory if the patient complains of:
1. Loss of consciousness
2. Weakness
3. Urinary incontinence
4. Dementia
E 27.Sensory examination includes the following:
1. Crude touch
2. Position
3. Vibration
4. Temperature
B 28. Bitemporal hemianopsia is secondary to:
1. Pituitary adenoma
2. Brainstem lesion
3. Uncal herniation
4. Metabolic encepalopathy
E 29. The appearance of the following reflexes would indicate frontal lobe lesion
1. Palmomental
2. Sucking
3. Grasp
4. Babinski
A 30. Babinski could be present if you have lesions of the:
1. Supratentorial
2. Posterior fossa
3. Spinal
4. Anterior horn cell
A 31. Neurologic examination in a comatose patient includes:
1. Light reflex
2. Fundoscopy
3. Mental status
4. Cerebellar exam
C 32. Retinal hemorrhage could be seen in:
1. Optic neuritis
2. Malignant
3. Cerebral infarction
4. Raptured aneurysm
hypertension
B 33. Mental status examination includes:
1. Level of consciousness 2. Memory
3. Mood
4. Position
B 34. Fundoscopy is valuable if you are entering:
1. Increased intracranial pressure 2. Demyelinating disorder 3. Raptures aneurysm 4. Cerebral infarction
C 35. Signs and symptoms of myopathy
1. Absent sensory changes
2. Proximal weakness
3. Atriphy
4. Hyporeflexia
A 36. Movement disorders associated with basal ganglia lesion
1. Athetosis
2. Ballismus
3. Parkinsons disease
4. Dystonia
C 37. Small, pinpoint pupils are associated with:
1. Amphetamines
2. Opiate overdose
3. Uncal herniation
4. Pontine lesion
overdose
D 38. Large, dilated pupils are associated with:
1. Cocaine use
2. Heroine use
3. Opiate overdose
4. Amphetamine overdose
E 39. Horners syndrome:
1. Mitotic pupils
2. Ptosis
3. Facial anhydrosis
4. Dilated pupils


D 40. Melkensson syndrome
1. Painful edema
2. Caused by herpes zoster virus 3. Pitting edema

C. MATCHING TYPE
I
41. Phonation
A. Cranial I
G
42. Numbness of the face
B. Cranial II
B
43. Visual acuity
C. Cranial III
E
44. Clenching of teeth
D. Cranial IV
K
45. Shoulder shrug
E. Cranial V
L
46. Tongue atrophy
F. Cranial VI
C
47. Convergence
G. Cranial VII


H. Cranial VIII


I. Cranial IX


J. Cranial X


K. Cranial XI


L. Cranial XII

B
48. CN II III
A. Corneal reflex
49. CN V VII
B. Pupillary light reflex
A
50.
CN
I
X


X

C. Gag reflex
C





























4.

Scrotal tongue

HEENT

1. Unilateral headache that can be localized behind the eyes CLUSTER
2. Enlarged blind spot occurs in a condition affecting the optic nerve
3. Rhinoscopy (ndi dapat tamaan) NASAL SEPTUM
4. polyps MEDIAL MEATUS
5. Family history MIGRAINE
6. Hyperthyroidism GRAVES DISEASE
7. button like CHANCRE IN SYPHILIS
8. maplike GEOGRAPHICAL
9. caused by deficiency in riboflavin and niacin SMOOTH TONGUE
10. ear pull (adults) UP & BACK
11. unilateral painless RETINAL WALL
12. examining the oropharynx use a tongue depressor DISTAL HALF OF TONGUE
13. white optic disc and tiny vessel are absent OPTIC ATROPHY
14. fissured tongue APPEARED WITH INCREASING AGE
15. nutritional deficiency (cold sore) ANGULAR CHEILITIS
16. caused by trauma SUBCONJUNCTIVAL HEMORRHAGE
17. tonsilar LN pulsation

a. carotid artery

b. ext. Jugular Vein

c. lymph adenopathy

d. bruit

18. convergence test
19. Tug test painful in OTITIS EXTERNA

ADDITIONAL:

Otitis Externa pale, moist , narrow

Retracted eardrum

Soft nodule in the thyroid gland GRAVEs

Behind the eye Cluster

Riboflavin def. and chemotx. smooth

Exposure to sumlight lip carcinoma

Factor to carcinoma Actinic Cheilitis

Worsen in noisy environmanet sensorineural hearing loss(SNHL)

Presbyopic better seen further away


Central loss

Button like infection Angular cheilitis

Ill fitting dentures

Fissure tongue increasing age

Sign of lip canar? Actinic cheilitis



1. Hold target at the midline and at eye level gradually moving the target toward the bridge of the nose.

a. Lid lag

b. Convergence

c. Accommodation

d. Confrontation

2. For the test above

a. This is normally maintained at a distance of 2-3 inches from the nasal bridge of the nose

b. Watch for the appearance of white sclera between the iris and the upper lid.

c. Usually a person sees both sets of fingers at the same time.

d. There is pupillary constriction in the opposite eye

3. There is poor convergence in

a. Hypothyroidism

b. Hypertension

c. Hyperthyroidism

d. Diabetes mellitus

4. Absent red reflex suggest

a. Normal eye

b. Artificial eye

c. Papilledema

d. Hyperthyroidism

5. External auditory canal is often swollen , narrowed moist, pale and tender. It may be reddened. This is

a. Chronic otitis externa

b. Acute otitis externa

c. Acute purulent otitis media

d. acute otitis media

6. This is not a special eye technique for eye examination

a. Nasolacrimal duct obstruction

b. Inspection of the upper palpebral conjunctiva

c. For assessing protruding eyes

d. Extraocular muscle test
7. Inspection of the nasal cavity through the anterior naris us usually limited to the following:

a. Vestibule

b. Superior turbinates

c. Sphenoid sinus

d. Frontal sinus
8. Spinning sensation is accompanied by nystagmus and ataxia

a. Vertigo

b. Dizziness

c. Tinnitus

d. Disequilibrium
9. Retracted tympanic membrane

a. More conical

b. Seen on Acute suppurative otitis media

c. Loss of bony landmarks

d. Accentuated bony landmarks

10. Local cause of nosebleeding



a. Flying

b. Hypertension

c. Nose Picking

d. Leukemia

e. English

11. Deacrease facial mobility and characteristic stare (Parkinsons disease)
12. Head is elongated with bony prominence of the forehead, nose and lower jaw (Acromegaly)
13. swelling usually appears first in the eyes and in the morning (Nephrotic syndrome)
14. hair is dry, coarse and sparse with periorbital edema. Lateral eyebrows thin (Myxedema)
15. Red cheeks, hirsutism and moonface (Cushings syndrome)
16. May accompany lipid disorders (Xanthelasma)
17. Tearing is prominent. Nasolacrimal duct obstruction is also noted (Dacryocyctitis)
18. Usually points inside the lid rather that the lid margin (Chalazion)
19. A painful, tender red infection in a gland at the margin of the eyelids (Sty)
20. Drooping of eyelids (Ptosis)

Q: Unilateral headache that can be localized behind the eye
A. Cluster

Q: Enlarge Blind spot occurs in a condition affecting optic nerve
A:

Q: Rhinoscopy (hindi dapat tamaan)
A: Nasal septum

Q: Polyps
A: Medial Meatus

Q: Family History
A: Migraine

Q: Hyperthyroidism
A: Graves disease

Q: Button like
A: Chancre syphilis

Q: Map-like
A: Geographical

Q: Caused by a deficiency in riboflavin and Niacin
A: smooth Tongue

Q: Ear pull (adult)
A: Up and back

Q: Unilateral, painless
A: Retinal Wall

Q: Examining oropharynx, use tongue depressor:
A: Distal half of tongue

10

11

Q: White optic disc and tiny vessel are absent


A: Optic Atropy

Q: Fissured tongue
A: Appeared with increasing age



Subarachnoid Hemorrhage - nausea and vomiting, possible loss of consciousness, neck pain

Ptosis - cause myasthenia gravis, damage to the oculomotor nerve (CN III), damage to the sympathetic nerve
supply ( Horner's syndrome)

Ectropion - the margin of the lower lid is turned outward, exposing the palpebral conjunctiva

Inflammation of the Lacrimal Sac (Dacryocystitis) - swelling between the lower eyelid and the nose

Horner's Syndrome- small affected pupil, reacts briskly to light and near effort, ptosis present, loss of
sweating on forehead, heterochromia

Argyll-Robertson pupils - small, irregular pupils that accomodate but do not react to light indicate CNS syphilis

Normal arterial wall is transparent; Normal light reflex is narrow

Silver wiring - occasionally a portion of a narrowed artery develops such as an opaque wall that has no blood
is visible within it.

Microaneurysms - tiny, round red spots seen commonly but not exclusively in and around the macular area;
minute dilatations of very small retinal vessels, but the vascular connections are too small to be seen
opthalmoscopically

Neovascularizations - formation of new blood vessels; more numerous, more tortuous, and narrower than
other blood vessels in the area and form disorderly looking red arcades

Hypertensive Retinopathy - marked arteriolar venous crossing changes are seen, copper wiring of the
arterioles is present. Cotton wool spot is seen just superior to the disc.

Nonproliferative Retinopathy ( Moderately severe) - tiny red dots/ microaneurysms

Proliferative Retinopathy ( Neovascularization) - new preretinal vessels arising on the disc extening across the
disc margins. Visual acuity is still normal, but risk for visual loss is high

Keloid - a firm, nodular, hypertrophic mass of scar tissue (binding) extending beyond the area of injury

Tophi - deposit of uric acid crystals characteristic of chronic tophaceous gout

Cutaneous cyst/ Sebaceous cyst - a dome shaped lump in the dermis forms a benign closed firm sac
attached to the dermis

Rheumatoid Nodules - small lump on the helix/antihelix and additional nodules elsewhere on the hands
along the surface of the ulna distal to the elbow

12

Acute Otitis Media with Purulent Effusion - caused by bacterial infection earache, fever and hearing loss.
Hearing loss is of the conductive type

Sensorineural loss - weber's test: sound lateralizes to good ear

Angular Cheilitis - softening of the skin at the angles of the mouth, fissuring

Chancre of Syphilis - appear on the lip, firm button-like lesion

Large Normal Tonsils - normal tonsils may be enlarged; protrude medially beyond the pillars and even to
the midline

Diptheria - dull red, gray exudate (pseudomembrane) is present on the uvula, pharynx and tongue

Koplik's spots - early sign of measles, small white specks that resembles grains of salt

Acute Necrotizing Ulcerative Gingivitis - ulcers develop in the interdental papilla

Hutchinson's teeth - sides of these teeth show normal contours; sides, shaping of the teeth are unaffected

Smooth tongue (Atrophic Glossitis) - lost its papillae, deficiency in riboflavin, niacin, folic acid, Vit. B12,
pyridoxine, iron

Apthous ulcer (Canker sores) - painful, round/oval ulcer that is white/yellowish gray and surrounded by a
halo of reddened mucosa

Diffuse Enlargement - endemic goiter

Hyperthyroidism - tachycardia

1.
2.

Enlarged skull may signify: Hydrocephalus or Pagets disease of Bone


20/200 vision meaning: at 20 ft., the patient can read print that a person with normal vision could read at
200 feet.
Absence of a red reflex: Cataract (opacity of lens), detached retina, retinoblastoma
Light rays from a distance focus on the anterior of retina: Myopia
Light rays from a distance focus on the posterior of retina: Hyperopia
Loss of venous pulsation in pathologic conditions like head trauma, meningitis, or mass lesions may be an
early sign of: Elevated ICP
Canal is swollen, narrowed, moist, pale, tender, reddened: Acute Otitis externa
Unilateral Conductive hearing loss: Sound is heard in the Impaired ear
Unilateral Sensorineural Hearing loss: Sound is heard in the good ear
Conductive hearing loss: BC > AC
Mucosa is reddened and swollen: Viral rhinitis
Mucosa is pale, bluish, or red: Allergic rhinitis
Submental Lymph node (maytanong bout sa Submental LN)
Basic landmark for palpating Thyroid gland: Thyroid cartilage and Cricoid cartilage??
Nausea, Vomiting: Migraine/Subarachnoid
Sudden movements of the head may be associated with: Brain tumor
Spinning sensation: Vertigo
Left Homonymous Hemianopsia: Right Optic Radiation
Damage to Oculomotor nerve: Ptosis
Eye no longer drains satisfactorily: Ectropion

3.
4.
5.
6.
7.
8.
9.
10.
11.
12.
13.
14.
15.
16.
17.
18.
19.

13
20. Bilateral Exopthalmos: Graves, hyperthyroidism
Unilateral exophthalmos: Graves dse/ Tumor/ Inflammation of the orbit
21. Painful, Tender, Red infection of the margin of the eyelid: Stye
22. Pupil is large, regular and usually unilateral; reaction to light is severely reduced/ slowed or absent: Adies
pupil (Tonic pupil)
23. Loss of venous pulsation; disc vessels more visible, more numerous: papilledema
24. Arteries show areas of focal/generalized narrowing: hypertension
25. Arteries become full and somewhat tortuous; Inc light reflex: Copper wiring
26. Presence of cotton-wool spot: Hypertensive Retinopathy
27. May tanong on page 267, di ko maalala pero meron
28. Softening of the skin at the angles of the mouth followed by fissuring: Angular Cheilitis
29. Firm lesion on the lip: Chancre of Syphilis
30. Reddened throat without exudate: Pharyngitis
31. Smooth tongue that has lost its papillae: Atrophic glossitis
32. Headache is severe and sudden onset: Subarachnoid hemorrhage/ Acute meningitis
33. Sudden unilateral visual loss is pinless: Retinal detachment/ retinal vein occlusion/ central retinal artery
occlusion/ vitreous hemorrhage/ macular degeneration
*If visual loss is painful: corneal ulcer/ uveitis/ acute glaucoma/ optic neuritis
34. Bilateral and painless visual loss: d/t cholinergics, anticholinergics and steroids/ Chemical, radiation
exposure
35. People having trouble understanding speech; noisy environment makes hearing worse: Sensorineural loss
36. Local cause of epistaxis: Trauma (nose picking) most common
37. Enlarged blind spot: Glaucoma/ Optic neuritis/ Papilledema



HEENT
1. In primary position, R eye deviates laterally but cannot move medially
a. R lateral rectus palsy
b. R oblique rectus palsy
c. R medial rectus palsy
d. R superior rectus palsy

2. Renal artery of HTN- focal narrowing

3. A portion of a narrowed artery develops such an opaque wall that no blood is visible with in it
a. silver artery or silver wire artery
b. copper wire
c. normal artery
d. retinal sclerosis

4. most important attribute for head ache.
A. Chronologic pattern
B. Quality
C. Location
D. Timing

5. Holding a pencil and moving toward the bridge of the nose (ganyan hung thought nung question, mahaba
kasi yung question e) Ans: Convergence test

14
6.
a.
b.
c.
d.
7.
8.
9.
10.
11.
12.

13.

14.
15.

16.
17.
18.
19.
20.








True about the test above


5-8 cm distance from the nose.









Inspection of anterior nares is limited to:
a. Vestibule
b. Sphenoid sinus
c. Sup. Turbinate
d. Frontal sinus

Spinning sensation with nystagmus and ataxia
a. Vertigo
b. Dizziness
c. Tinnitus
d. Dysequilibrium
ans. A. VERTIGO?

Retracted tympanic membrane.

Local cause of nose bleeding
A. Flying
B.Hypertension
C. Nose picking
D. Leukemia

In opthalmoscopic examination,-The view is limited to posterior structure

Benign lesion associated with antibiotic therapy- hairy tongue
Examination of LN is done by- palpation
Basic landmark of thyroid gland- Cricothyroid


Test I: Choose the BEST answer
1. In opthalmoscopic Examination
a. The view is limited to posterior structure
b. Pheripheral structures can be evaluated in the absence of mydriatic drops
c. (+)3 or (+)4 diopters will allow you to see the anterior structures clearly
d. Red-orange reflex is normally not visualize first

2. A (-) lens is used in
a. Hyperopic eyes
b. Aphakic eyes
c. Myopic eye
d. Astigmatism

3. Physical sign of retrosternal goiter
a. Venous engorgement
b. Tender thyroid
c. Thyroid bruit
d. Regional lymphnode enlargement

4. Examinatin of the lymphnode is primarily by
a. Inspection
b. Palpation
c. Auscultation
d. X-ray

5. Basic landmark for thyroid gland examination
a. Cricoid cartilage
b. Thyroid cartilage
c. Trachea
d. Sternocleidomastoid

6. Primary lesion from posterior 2/3 of the scalp and nasopharynx
a. Submental lymphnodes
b. Posterior cervical triangle
c. Anterior crvical triangle
d. Supraclavicular lymphnode

7. Nasal flaring is associated with
a. Respiratory distress
b. Chronic nasal obstruction

15


c. Mouth breathing
d. Allergic rhinitis

8. A hole in nasal septum is commonly caused by
a. Syphilis
b. Tuberculosis
c. Cocaine abuse
d. Repeated trauma in picking off crusts

9. Headache presents on aakening
a. Migraine
b. Brain tumor headache
c. Meningitis
d. Subarachnoid hemorrhage

10. Sudden unilateral painless visual loss
a. Acute glaucoma
b. Corneal ulcer
c. Uveitis
d. Retinal vein occlusion

Test II: Match the abnormalities of the lips with description below
a. Angular cheilitis
b. Cold sore
c. Chancre of syphilis
d. Carcinoma of the lips
e. Angioedema

B 11. Produce recurrent and painful eruptions of the lips and surrounding skin
A 12. Maybe due to ill-fitting dentures
C 13. Highly infectious, firm, button-like lesion that ulcerates and may become crusted
D 14. Fair skin and prolonged exposure to the sun are common risks factors
A 15. It may be due to nutritional deficiency

Test III: Match the pattern of hearing loss with description below
a. Conductive hearing loss
b. Sensorineural hearing loss

A 16. One cause is otitis media
B 17. In weber test, sound lateralizes to good ear

16


B 18. In Rinne test, normal pattern prevails
A 19. Voice maybe loud because hearing is difficult
A 20. Usual ageof onset childhood and adulthood, up to age 40




































17

18

HEENT
a. Conductive hearing loss
b. Sensorineural hearing loss

1.

Voice may be loud because the patient has trouble hearing his or her own voice B

2.

Age of onset is most often in childhood and young adulthood, up to age 40 A

3.

Otitis media A

4.

Sound lateral to good ear in Weber test B

5.

AC>BC, Rinne test B


6.

Basic landmark cricoid cartilage

7.

Lesion in the posterior 2/3 of the scalp and nasopharynx subscapular

8.

Retrosternal goiter venous engorgement

9.

Examination of the lymphnode palpation

10. Ophalmoscopic examination


a. The view is limited to posterior structure
b. Pheripheral structures can be evaluated in the absence of mydriatic drops
c. (+)3 or (+)4 diopters will allow you to see the anterior structures clearly
d. Red-orange reflex is normally not visualize first
11. A (-) lens is used in
a. Hyperopic eyes
b. Aphakic eyes
c. Myopic eye
d. Astigmatism

12. Firm, button-like lesion chancre of syphilis
13. Risk factors are fair skin, and prolonged exposure to sun carcinoma of the lip
14. Nutritional insufficiency angular cheilitis
15. Ill-fitting dentures angular cheilitis
16. Painful vesicular lesions in angle of the mouth herpes simplex/cold sore
17. Unilateral, painless visual loss
a. Acute glaucoma
b. uveitis
c. retinal vein occlusion
d. all of the above
18. Headache upon waking up migraine
19. Nasal flaring associated to
a. Respiratory distress
b. Chronic nasal obstruction
c. Mouth breathing

19
d. Allergic rhinitis
20. Hole in the basal septum most common in
a. Syphilis
b. Tuberculosis
c. Cocaine abuse
d. Repeated trauma in picking off crusts


































Test I: Choose the BEST answer
1. In opthalmoscopic Examination
a. The view is limited to posterior structure
b. Pheripheral structures can be evaluated in the absence of mydriatic drops
c. (+)3 or (+)4 diopters will allow you to see the anterior structures clearly
d. Red-orange reflex is normally not visualize first

2. A (-) lens is used in
a. Hyperopic eyes
b. Aphakic eyes
c. Myopic eye
d. Astigmatism

3. Physical sign of retrosternal goiter
a. Venous engorgement
b. Tender thyroid
c. Thyroid bruit
d. Regional lymphnode enlargement

4. Examinatin of the lymphnode is primarily by
a. Inspection
b. Palpation
c. Auscultation
d. X-ray

5. Basic landmark for thyroid gland examination
a. Cricoid cartilage
b. Thyroid cartilage
c. Trachea
d. Sternocleidomastoid

6. Primary lesion from posterior 2/3 of the scalp and nasopharynx
a. Submental lymphnodes
b. Posterior cervical triangle
c. Anterior crvical triangle
d. Supraclavicular lymphnode



20


7. Nasal flaring is associated with
a. Respiratory distress
b. Chronic nasal obstruction
c. Mouth breathing
d. Allergic rhinitis

8. A hole in nasal septum is commonly caused by
a. Syphilis
b. Tuberculosis
c. Cocaine abuse
d. Repeated trauma in picking off crusts

9. Headache presents on aakening
a. Migraine
b. Brain tumor headache
c. Meningitis
d. Subarachnoid hemorrhage

10. Sudden unilateral painless visual loss
a. Acute glaucoma
b. Corneal ulcer
c. Uveitis
d. Retinal vein occlusion

Test II: Match the abnormalities of the lips with description below
a. Angular cheilitis
b. Cold sore
c. Chancre of syphilis
d. Carcinoma of the lips
e. Angioedema

B 11. Produce recurrent and painful eruptions of the lips and surrounding skin
A 12. Maybe due to ill-fitting dentures
C 13. Highly infectious, firm, button-like lesion that ulcerates and may become crusted
D 14. Fair skin and prolonged exposure to the sun are common risks factors
A 15. It may be due to nutritional deficiency



21


Test III: Match the pattern of hearing loss with description below
a. Conductive hearing loss
b. Sensorineural hearing loss

A 16. One cause is otitis media
B 17. In weber test, sound lateralizes to good ear
B 18. In Rinne test, normal pattern prevails
A 19. Voice maybe loud because hearing is difficult
A 20. Usual ageof onset childhood and adulthood, up to age 40






























22

23

HEENT
a. Conductive hearing loss
b. Sensorineural hearing loss

1.

Voice may be loud because the patient has trouble hearing his or her own voice B

2.

Age of onset is most often in childhood and young adulthood, up to age 40 A

3.

Otitis media A

4.

Sound lateral to good ear in Weber test B

5.

AC>BC, Rinne test B


6.

Basic landmark cricoid cartilage

7.

Lesion in the posterior 2/3 of the scalp and nasopharynx subscapular

8.

Retrosternal goiter venous engorgement

9.

Examination of the lymphnode palpation

10. Ophalmoscopic examination


a. The view is limited to posterior structure
b. Pheripheral structures can be evaluated in the absence of mydriatic drops
c. (+)3 or (+)4 diopters will allow you to see the anterior structures clearly
d. Red-orange reflex is normally not visualize first
11. A (-) lens is used in
a. Hyperopic eyes
b. Aphakic eyes
c. Myopic eye
d. Astigmatism

12. Firm, button-like lesion chancre of syphilis
13. Risk factors are fair skin, and prolonged exposure to sun carcinoma of the lip
14. Nutritional insufficiency angular cheilitis
15. Ill-fitting dentures angular cheilitis
16. Painful vesicular lesions in angle of the mouth herpes simplex/cold sore
17. Unilateral, painless visual loss
a. Acute glaucoma
b. uveitis
c. retinal vein occlusion
d. all of the above
18. Headache upon waking up migraine
19. Nasal flaring associated to
a. Respiratory distress
b. Chronic nasal obstruction
c. Mouth breathing

24
d. Allergic rhinitis
20. Hole in the basal septum most common in
a. Syphilis
b. Tuberculosis
c. Cocaine abuse
d. Repeated trauma in picking off crusts



Our Lady of Fatima University
College of Medicine Regular class
nd
2 semester 2013-2014
Clinical Medicine
HEENT
Name: _____________________________________ Section: ______________ Date: _________ Score:______

1. Major attributes to headache
a. Location
c. Chronological pattern

b. Quality
2. Headache is episodic and tends to peak several hours. This is:

a. Migraine

c. brain tumor

b. Cluster headache
d. meningitis
3. Nausea and vomiting is common in:

a. Meningtis

c. Brain tumor

b. Tension headache
d. Rebound
4. Cough, sneezing ang changing patternof the head can increase the pain from:

a. tension headache
c. Brain tumor

Meningitis/migraine
d. cluster headache
5. Aging vision: Presbyopia
6. Bilateral bilateral painful eye maybe due to the following

a. Chronic radiation exposure
c. Central retinal occlusion

b. Cholinergic medication
d. corneal ulcer
7. Horizontal diplopia

a. weakness or paralysis of EOM
c. lesion CN III or CN V

b. Lesion CN III or IV


d. problems in cornea
8. In conductive hearing loss

a. Have particular trouble understanding speech

b. Noisy environment makes hearing worse

c. Problem in inner ear

d. noisy environment help
9. perception that the patient or the environment is rotating or spinning. This is:

a. Tinnitus

b. Vertigo

c. Dizziness

d. Menieres disease
10. perceived sound that has no external stimulus: a. dizziness
11. sensation of spinning: Vertigo
12. Fever, pharyngeal exudates, anterior lymphadenopathy without cough:

a. viral pharyngitis

c. Diptheria

b. Strep Pharyngitis

d. Infectious Mononucleosis
13. Hyperthyroidism

a. Cold intolerance


c. Weight loss

b. Preference of warm clothing
d. Decreased sweating
nd
14. Cause of blindness in African American and 2 leading cause of blindness overall?
a. Cataract

c. weight loss

b. Glaucoma

d. decreased sweating
15. 20/40 corrected is:

a. The patient can read the line 40 with glasses

b. Patient can read the line without glasses

c. Vision is normal

d. Patient is presbyopic

25


16. An enlarged blind spot occurs in condition affecting optic nerve such as:

a. Optic atrophy
c. Glaucoma

b. Cataract

d. Papilledema
17. Contraindication for mydriatic drops

a. Coma

c. Arcus senillis

b. Cataract

d. pterygium
18. The Tug test is painful in:

a. Otitis media
c. chronic otitis externa

b. Acute otitis externa
d. purulent otitis media
19. Unilateral sensory neural loss

a. Sound is heard at the good ear

b. Sound is heard in the impaired ear

c. Due to impact cerumen

d. due to eardrum perforation
20. The nasal mucosa is pale, bluis or red. This suggests:

a. Viral rhinitis
c. allergic rhinitis

b. Acute sinusitis
d. normal mucosa
21. A triangular thickening of the bulbar conjunctiva: a. pterygium
22. Characteristic of Optic atrophy

a. Visible optic vessel
c. absent optic vessel

b. tiny optic vessel
d. no vessel pulsation
23. Normal retinal artery

a. Arterial wall transplant

c. Narrow light reflex

b. narrow column
of blood vessel
d. focal narrowing
24. Fullness and popping sound in the ear with mild conductive hearing loss and ear pain

a. Acute otitis media

c. with effusion

b. Chronic otitis externa
25. Bulging eardrum

a. Hearing loss is sensorineural
c. accentuated

b. Obscured


d. changes in atmospheric pressure
26. The patient complains of earache and hearing loss. The eardrum are reddened. Losses its landmark and
buldges laterally towards the examiners eye

a. Acute otitis media with purulent effusion

b. Chronic otitis externa

c. Chronic otitis media

d. Acute otitis externa
27. The skin of the ear canal is often thickened, red and itchy: Chronic otitis externa
28. Button like lesion:
Chancre syphilis
29. Benign condition that may follow antibiotic therapy:

a. Geographic tongue
c. smooth tongue

b. Fissured tongue
d. hairy tongue
30. Deficiency in riboflavin, niacin, folicacid, B12 and pyrodoxin

a. Atrophic

c. Hairy

b. Fissured

d. Geographic
31. Basic landmark of the thyroid

a. Thyroid cartilage
c. Trachea

b. Cricoid cartilage
d. Isthmus
32. Furosemide medication that affect the hearing
33. Nose picking for local cause of epistaxis
34. Rhinitis medicamentosa excessive use of decongestant
35. Cause of excessive tearing
36. Absence of red reflex
37. Bilateral/unilateral exopthalmus

26

27

38. Actinic cheilitis


39. Diphtheria
40 Canker sore a painful round or ovalulcer that is white or yellowish gray that is surrounded by halo reddened
mucosa
41. Tophi deposits of uric acid crystals
















































1.

2.

3.
4.

5.

6.
7.

8.
9.

10.
11.

12.

13.
14.

15.

28

Vision of 20/200 means that


a. at 20 ft, the patient can read print that a person with normal vision could read at 200 ft
b. at 200 ft, the patient can read print that a person with normal vision could read at 20 ft.
c. the larger the first number, the worse the vision
d. normal vision
An image from the upper nasal visual field strikes the
a. Upper temporal area
c. Lower temporal area
b. Lower nasal area
d. Upper nasal area
Sees better when the card is farther away
a. Nearsightednesss
c. Presbyopia
b. Myopia
d. hyperopia
Absence of red reflex indicates
a. Artificial eye
c.
b. Opacity of lens
d.
An enlarged blind spot occurs in
a. Graves
c. Optic neuritis
b. Retroorbital tumor
d. Diabetic neuropathy
Headache from errors of refraction include
a. nearsightedness
c. astigmatism
b. farsightedness
d. myopia
Testing near reaction is used in diagnosis of
a. Argyll Robertson pupil
c. Oculomotor nerve paralysis
b. Anisocoria
d. Horners syndrome
Fixed defects (scotoma) are seen in
a. Retina
c. Lens
b. Cornea
d. Pupil
Excessive tearing from increased production is due to
a. Corneal irritation
c. Entropion
b. Extropion
d. Nasolacrimal duct obstruction
Most important attribute of headache
a. Severity
c. Quality
b. Chronological pattern
d. location
Hyperthyroidism
a. Intolerance to cold
c. Involuntary weight loss
b. Preference for warm clothing
d. Decreased sweating
The tug test is painful in
a. Otitis media
c. Chronic otitis externa
b. Acute otitis externa
d. Purulent otitis media
Bilateral, painless change in refractory may be due to
a. Chemical exposure
c. Steroids
b. Radiation exposure
d. Diabetes
Family history may be positive is
a. Tension
c. Migraine
b. Cluster
d. Medication obveruse
The eardrum itself is scarred, no landmarks visible, often closes in healing processs
a. Tympanosclerosis
c. Otosclerosis
b. Perforation of eardrum
d. Serous effusion


Retinal Arteries and Arteriovenous Crossing

C
16. Arteries may show focal or generalized narrowing
B
17. Opaque wall and no blood visible

a. Copper wore
b. Silver wire

29

A
18. Arteries close to the disc become full and somewhat torturous
c. Retinal Arteries in Hypertension
C
19. Arterial wall is invisible A-V crossing is visible


d. Normal Retinal Artery
C
20. Light reflex is narrow about one-fourth the diameter of blood column

Pupillary Abnormalities

D
21. pupils that accommodate but do not react to light


a. Anisocoria
C
22. dialted pupil is fixed to light and near effort


b. Adies pupil
B
23. slow accommodation causes blurred vision


c. Oculomotor nerve paralysis
B
24.pupil is large regular, usually unilateral


d. Argyll Robertson pupil
A
25.causes include blunt trauma to the eyes, open-angle glaucoma


Diplopia

A
26. Images are side by side



a. Horizontal diplopia
A
27. Caused by palsy of CN III or IV

b. Vertical diplopia
D
28. Diplopia in one eye with the other closed


c. Both
B
29. Images are on top of each other



d. None
A
30. One kind of this diplopia is physiologic

30


Hearing Loss

A
31. Abnormality is usually visible except in the otosclerosis
a. Conductive hearing loss
B
32. In Weber test, room noise not appreciated


b. Sensorinueral hearing loss
B
33. Voice is loud because hearing is difficult


c. Both
C
34. To estimate hearing, test one ear at a time


d. None
B
35. In Rinne test, the normal pattern prevails

Eardrum

D
36. Pink, grayish, intact



a. Serous Effusion
C
37. Caused by acute purulent infection of the middle ear
b. Perforated Eardrum
B
38. Eardrum itself is scarred, no landmarks are visible
c. Acute Otits Media with Purulent Effusiion
A
39. Amber fluidbehind the eardrum is characteristic.
d. Normal Eardrum
Air bubbles can be seen with the amber fluid
C
40. Eardrum is bulging and redden, most landmarks are obscured

Clinical Findings

D
41. Diptheria




a. Benign midline lump
F
42. Apthous ulcer



b. Nasal sputum
G
43. Epulis




c. Poor convergence
E
44. Fever blister



d. Pseudomembrane
J
45. Microaneurysm



e. Cold sore
H
46. Sentinel node



f. Canker sore
B
47. Transilumination



g. Pregnancy tumor
I
48. Hyperthyroidism



h. Thoracic malignancy
K
49. Hypothyroidism



i. (+) lidlag
A
50. Torus palatus



j. Diabetes







k. None of the above


1. Major attributes to headache
a. Location
b. Quality
c. Chronologic pattern
d. Hindi kya AOTA to? Kasi tama lahat
2. Headache is episodic and tends to peak after several hours.
a. Migraine
b. Cluster
c. Brain tumor
d. Meningitis
3. Nausea and vomiting is common in
(MIGRAINE ang sagot, wala s choices. Lahat ng choices walang assoc. nausea and vomiting)
a. Meningitis
b. Tension
c. Brain tumor
d. Rebound

31


4.

5.
6.

7.

8.

9.

10.
11.
12.

13.

14.

Cough, sneezing, changing position of the head can increase the pain from:
a. Tension headache sustained muscle tension
b. Migraine noise and bright lights
c. Brain tumor (and sinusitis)
d. Cluster alcohol
Aging vision PRESBYOPIA
Bilateral painful eye
a. Chronic radiation exposure
b. Cholinergic medication bilateral, painless
c. Central retinal artery occlusion unilateral, painless
d. Corneal ulcer- unilateral, painful
Horizontal diplopia
a. Weakness or paralysis of EOM
b. Lesion of CN III or IV
c. Lesion of CN III or VI
d. Corneal problems
Conductive hearing loss
a. Have particular trouble understanding speech sensorineural
b. Noisy environment makes hearing worse sensorineural
c. Problem in inner ear sensorineural
d. Noisy environment may help
Perception that the patient or the environment is spinning\
a. Tinnitus perceived sound without external stimulus
b. Vertigo
c. Dizziness
d. Menieres disease tinnitus + hearing loss + vertigo
Perceived sound without external stimulus TINNITUS
Sensation of spinning VERTIGO (hindi kaya dizziness to?)
Fever, pharyngeal exudates, anterior lymphadenopathy, no cough
a. Viral pharyngitis
b. Strep pharyngitis
c. Diphtheria
d. Infectious mononucleosis
Hyperthyroidism
a. Cold intolerance hypo
b. Preference of warm clothing hypo
c. Weight loss
d. Decrease sweating hypo
nd
Leading cause of blindness in African American and 2 leading cause of blindness overall
a. Cataract
b. Glaucoma
c. Macular degeneration
d. Retinal detachment

32


15. 20/40 corrected is
a. Patient can read line 40 with glasses
b. Patient can read the line without glasses
c. Vision is normal
d. Presbyopia
16. An enlarged blind spot occurs in conditions effecting the optic nerve such as: (glaucoma, optic neuritis,
papilledema)
a. Optic atrophy
b. Cataract
c. Glaucoma
d. Papilledema
17. Contraindications for mydriatic drops (head injury, coma, suspicion of narrow angle glaucoma)
a. Coma
b. Cataract
c. Arcus senilis
d. Pterygium
18. Tug test is painful in
a. Otitis media
b. Acute otitis externa
c. Chronic otitis externa
d. Purulent otitis media
19. Unilateral sensorineural loss
a. Sound is heard in good ear
b. Sound is heard in impaired ear unilateral conductive
c. Impacted cerumen
d. Eardrum perforation
20. The nasal mucosa is pale, bluish, or red
a. Viral rhinitis
b. Acute sinusitis
c. Allergic rhinitis
d. Normal mucosa
21. Triangular thickening of bulbar conjunctiva PTERYGIUM
22. Characteristic of optic atrophy
a. Visible optic vessel
b. Tiny optic vessel
c. Absent optic vessel
d. No vascular pulsation
23. Normal retinal artery
a. Arterial wall is transparent
b. Narrow column of blood vessel HPN
c. Narrow light reflex
d. Focal narrowing HPN
24. Fullness and popping sound in the ear with mild conductive hearing loss and ear pain SEROUS EFFUSION
25. Bulging eardrum (Parang wala tamang sagot. Dapat acute otitis media with purulent effusion)
a. Hearing loss is sensorineural conductive

33

26.

27.
28.
29.

30.

31.

b. Obscured
c. Accentuated
d. Changes in atmospheric pressure serous effusion
The patient complains of earache and hearing loss. The eardrums are reddened, loses its landmark, and
bulges lateral towards examiners eye.
a. acute otitis media with purulent effusion
b. chronic otitis media externa
c. chronic otitis media
d. acute otitis externa
skin of the ear canal is often thickened, red and itchy CHRONIC OTITIS EXTERNA
Button like lesion CHANCRE IN SYPHILLIS
Benign condition that may follow antibiotic therapy
a. Geographic tongue dorsum shows scattered smooth areas denuded of papillae
b. Fissured tongue increasing age
c. Smooth tongue / atrophic glossitis deficiency of riboflavin, niacin, folic acid, vitB12, pyridoxine, iron
or treatment with chemotherapy
d. Hairy tongue HIV, AIDS
Deficiency of riboflavin, niacin, folic acid, B12, pyridoxine
a. Atrophic glossitis
b. Fissured
c. Hairy
d. Geographic
Basic landmark of thyroid cartilage
a. Thyroid cartilage
b. Cricoid cartilage
c. Trachea
d. Isthmus


















34

EXTREMITIES
Clinical Medicine EXTREMITIES

1.
Test for nerve irritation..

a.
Trendelenberg HIP DISLOCATION (PAINLESS LIMPING)
b.
Faber Test
c.
Thomas Test HUGH OWEN THOMAS SIGN (TEST THIGH AND HIP JOINT) LUMBAR
PELVIC DISEASE, HIP JOINT DISEASE
d.
None

THOMAS SIGN- TEST FLEXION (FLATTENING OF LUMBAR CURVE IS OBSERVE)

2.
Type of primary joint articulation that allows free movement..

a.
Fibrous
b.
Synovial
c.
Cartilagenous
d.
All

3.
Manifestation / Cause of extreme dorsiflexion foot

a.
Ruptured Achilles tendon absence plantar flex (simmonds test)
b.
Soleus tear malformation leg severe pain and tenderness
c.
Gastrocnemius strain
d.
All

4.
Site of muscle of the rotator except

a.
Infraspinatus
b.
Supraspinatus
c.
Teres major
d.
None

5.
ROM of elbow
Ans. Flexion, extension, supination, pronation

6.
Anterior knee cyst INFLAMMATION OF THE BURSA

a.
Housemaid PREPATELLAR BURSITIS
b.
Clergyman INFRAPATELLAR BURSITIS
c.
Bakers cyst MORRANT BAKER POPLITEAL ARTERY ENTRAPMENT
d.
A &b

BAKER
- COMPLICATION OF RA

7.
Osteoarthritis

a.Heberdens nodes
b. Bouchards nodes
c. asymmetric..

LORDOSIS,


d. all

8.
Palmar erythema except

a.portal HPN
b.aplastic anemia
c. pregnancy
d.valvular heart dse


9.
Wrist ROM

Ans. Flexion, extension, abduction, adduction

10.
True regarding neck ROM

a.flexion and extensionat the skull and C1 C3-C7
b.rotation at c1 and c2 ATLANTO-AXIAL
c.a&b
d. none

11.
Visual examination from behind.. landmarks except

a.spinous process of T1 - PALPATION
b.iliac crest
C.POSTERIOR SUPERIOR ILIAC SPINE
d. dimple of venus

12.
Test for Lumbar lordosis
ANS. THOMAS TEST

13.
Examination of motor function

a.look for tremor
b.look for coordinated movement
c.muscle size
d.all

14.
Pronated hand dropped from wrist
a.
Carpal tunnel syndrome
b.
Ulnar nerve palsy
c.
Median nerve palsy (MEDIAN = RADIAL)
d.
None

15.
Lateral deviation from midline
ANS. GENU VALGUM




16.
Rheumatoid arthritis

A.MIP &POP INVOLVEMENT

35

36

B.HAYGARTHS NODES
c.Bouchards nodes - OA
d.all
e.a&b

17.
Unequal shoulder length

A.SCOLIOSIS
B.SPRENGELS DEFORMITY
c.a&b
d.none

18.
Unequal leg length except
Ans. Kyphosis

19.
Long narrow nails except

a.hypopotuitarism
b.eunochoidism
c. cretinism - SQUARE
d.none

20.
Lordosis except
Ans. A. convexity


NOTE: Kindly Recheck all the answers!

God bless!







37
A. Matching type

E 1. Drumstick fingers
C 2. Spider fingers
3. Subungal haemorrhage
A 4. Graenlen's test
H 5. Lesague's test
E 6. Hypertrophic osteoarthropathy
G 7. Trendelenburg's sign
8. Tinel' sign
K 9. Infrapatellar bursitis
10. Genu recurvatum
M 11. Acromegaly
F 12. Azure half-moon
D 13. Hypothenar
-----
-----



B. MULTIPLE CHOICE

16. Thumb in Filipino is:
A. Hinlalaki
B. Palasingsingan

17. Hinlalato is:
A. Thumb
B. Index finger

18. Severe carpal tunnel syndrome is managed by:
A. NSAIDs
B. Physical therapy

19. Not a malposture of the hand:
A. Claw hand
B. Ape hand

20. Most common hypothesis of clubbing:
A. Unknown mechanism
B. Hypoxia

21. Lateral bending of the spine:
A. Atlanto-axial joint
B. Midcervical vertebra

22. Eggshell nails:
A. Vitamin A deficiency
B. Hypochromic anemia

A. Passive hyperextension
B. Radial nerve injury
C. Arachnodactyly
D. Ulnar nerve injury
E. Clubbing
F. SBE
G. Hip dislocation
H. Straight leg test
I. Carpal tunnel syndrome
J. Housemaid's knee
K. Clergyman's knee
L. Small patella
M. Square & round nail plate
N. -----
O. -----
P. -----

C. Hinlalato
D. None

C. Middle finger
D. None
C. Complete rest of the wrist
D. Surgical decompression

C. Wrist drop
D. Polydactyly
C. Parrot beak nails
D. (?)

C. Atlanto-occipital
D. C3-C7
C. Thyrotoxicosis
D. Bronchiectasis


23. Brittle nails:
A. Onycholysis
C. Onychocrytosis
B. Onychorrhexis
D. Onychocryptosis

24. Ram's horn nails:
A. Onycholysis
C. Onychocrytosis
B. Onychorrhexis
D. Onychocryphosis

25. Fixation of the 2nd toe in flexion:
A. Hallus valgus
C. Hammer toe
B. Hallus rigidus
D. Callus

26. Point of reference of the fingers:
A. Wrist joint
C. Ulnar nerve
B. Middle finger
D. Radial nerve

27. Rheumatoid arthritis is mostly seen in
A. Fingers
C. Elbow
B. Knees
D. Hip joint

28. Pain referred to the shoulder:
A. Pneumonia
C. Winged scapula
B. Supraspinatus tendinitis
D. All

29. Excessive transverse growth of the nail plate causing lateral edge to lacerate:
A. Onychauxis
C. Ram's horn nails
B. Onychocryptosis
D. Onychogryphosis


30. One of the following regarding the examination of the spine is incorrect:
A. From the side inspect the spinal profile
B. From behind inspect the lateral curve
C. Percuss the spine with a neurological hammer
D. Use your thumb to palpate for the para vertebral area

31. One of the following is a cause of kyphosis in postmenopausal women:
A. Osteoporosis
C. Osteus deformans
B. Faulty posture
D. Ankylosing spondylitis

32. When the legs deviate towards the midline and the knees farther apart, it is called:
A. Genu varum
C. Genu recurvatum
B. Genu valgum
D. None of the above

33. Presence of blood in the joint cavity:
A. Hemarthrosis
C. Hemoptysis
B. Hematemesis
D. Hematochezia


34. The length of the lower extremities can be measured by:
A. ASIS to the tip of the medial malleolus with the tape crossing the patella
B. ASIS to the tip of lateral malleolus
C. From iliac crest to Achilles tendon
D. From iliac crest to medial malleolus

38



35. When doing the lumbar puncture, iliac crest is at:
A. 4th lumbar vertebrae
C. T12- L1
B. L1-L2
D. S4


36. Extreme dorsiflexion of the foot
A. Ruptured Achilles tendon
C. Soleus tear
B. Fracture of the tibial shaft
D. Fracture of the fibular shaft

37. Pressure diverticulum of the synovial sac protruding thru the joint capsule of the knee
A. Baker's cyst
C. Pyarthrosis
B. Popliteal abscess
D. Prepatellar bursitis

38. Swelling in the popliteal fossa
A. Baker's cyst
C. Prepatellar bursitis
B. Popliteal abscess
D. Infrapatellar bursitis

39. Which of the following is the most common cause of painful swelling in young males?
A. Paget's disease
B. Ankylosing spondylitis
C. Pott's disease
D. Osteoporosi

39




UPPER EXT
1. Hands (ROM ABDUCTION, ADDUCTION, EXTENSION, FLEXION)
Condition impaired ROM = FAT
1. Fibrosis of palmar fascia (DUPUYTRENS CONTRACTURE)
2. Arthritis
3. Tenosynovitis (inflammation of tendon sheaths)

Size hands
LONG
ACROMEGALY
AFTER EPIPHYSIS CLOSE (ADULT)
EXCESSIVE PRODUCTION GH
NOT SYMMETRICAL
GIGANTISM
BEFORE EPIPHYSIS CLOSE (CHILD)
PROPORTIONATE AND SYMMETRIC
ND
NOTE: BOTH ARE 2 TO TUMOR + BY AN EXCESS OF SOMATOTROPHIC HORMONE FROM EOSINOPHILIC ADENOMA OF
ANTERIOR PIT GLAND

LOND AND SLENDER HANDS
*SPIDER FINGERS
SMALL, THICK HANDS
CRETINISM (CHILD)
DWARF PX
MYXEDEMA (OLD)
ABSOLUTE DEF OF TH
HANDS SHORT, THICK, FAT
RADIUS MAY BE SHORTENED
MONGOLISM
CHROMOSOMAL ABN
HANDS SHORT, THICK, THUMB DIVERGES FROM
NEARER THE WRIST THAN NORMAL
LITTLE FINGER IS CURVED (RADIAL WARD)

MALPOSTURE (ABNORMALITY IN POSTURE) = CAW
CLAW HAND
BUSMA
BRACHIAL PLEXUS
ULNAR NERVE INJURIES
SYRINGOMYELIN
MUSCULAR ATROPHIES
ACUTE POLIOMYELITIS
APE HAND
PAS
PROGRESSIVE MUSCULAR ATROPHY
AMYOTROPHIC LATERAL SCLEROSIS
SYRINGOMYELIN
WRIST DROP
RPP
RADIAL NERVE INJURIES
POLIOMYELITIS
POISONING (LEAD, ARSENIC, ALCOHOL)
2. PALM
ABNORMALITIES

CAROTENODERMA
HEPATIC DISEASE
MYXEDEME
THENAR ATROPHY
MEDIAN NERVE
HYPOTHENAR ATROPHY
ULNAR NERVE
HANSENS
THICK AND CORD LIKE

40

PALMAR ERYTHEMA


3. FINGERS
MALFORMATION
POLYDACTYL (SUPERNUMENARY FINGER)

SYNDACTYL (WEBBED FINGERS)


HEBERDENS NODES (OSTEOARTHRITIS)

HAYGARTHS NODES (RHEUMATOID ARTHRITIS)


5. DIGITS
INFECTION
PARONYCHIA

FELON


6. FINGERNAILS
MALFORMATION

ONYCHORREXIS
- BRITTLE NAIL PLATE
- BORDER FRAYED AND TORN

CLUBBING W/O PERIOSTOSIS
- CONVEX
ND
- 2 HYPOXIA
- PARROT-HIPPOCRATIC-SERPENT-DRUMSTICK
ABSENCE OF NAILS
- CONGENITAL
BITTEN NAILS
- IRREGULAR
- SHORT NAILS
SQUARE AND ROUND

41
HYPOTHENAR = FLAT AND ATROPHIED
+NUMBNESS
LLV
LIVER CIRRHOSIS
LATE STAGE PREGNANCY
VULVAR HEART DISEASE

LAURENCE BIEDL SYNDROME


ASSOCIATED WITH:
1. JUVENILE OBESITY
2. RETINAL DEGENERATION
3. GENITAL HYPOPLASIA
4. MENTAL RETARDATION
CONGENITAL/HEREDITARY
OSTEOARTHRITIS
PAINLESS DIJ
HARD
OLD WOMEN
MEN (DUE TO TRAUMA, SINGLE JOINT)
INFLAMMATORY
MIDDLE AND PROXIMAL
FUSIFORM
PAINFUL
JOINT CAPSULE THICKENED

SWOLLEN
REDDENED
PAINFUL
ABSCESS TERMINAL PULP
ND
2 BACTERIAL INFECTION
ONSET SWELL AND DULL PAIN

MALNUTRITION
IRON DEFICIENCY
THYROTOXICOSIS
CALCIUM DEFICIENCY
TB
COPD
BROCHIECTASIS
ICHTHYOSIS
TRAUMA
PERSONALITY DISORDER
NEUROLOGIC
CAS
ACROMGELY


LONG AND NARROW

SPOON
- CONCAVE
- KOILONYCHOSIS

EGGSHELL
- CONCAVE
ONYCHAUXIS
- HYPERTHOPHY
- NAILS ON TOP OF ANOTHER
- IRREGULAR DISCOLORED
- UNKNOWN CAUSE
RED HAFT MOON
AZURE BLUE HAFT MOON
ONYCHOLYSIS
- SEPRATION OF NAIL FROM NAILBED

42
CRETINISM
HEM
HYPOPITIUTARISM
EUNOCHOIDISM
MARFANS SYNDROME
HIS
HYPOCHROMIC ANEMIA
IRON DEFICIENCY
RARE: RLS
- RHEUMATIC
- LICHEN PLANUS
- SYPHILIS
VIT A DEFICIENCY
ND

2 TO FUNAL INFECTION
WASHER WOMEN

CARDIAC
LANULA RED
WILSON
LANULA BLUE
ND
2 TO FUNAL INFECTION AND TRAUMA
+FOUL ODOR
- THYROTOXICOSIS
- ECZEMA
- PSORIASIS
- MYENTERIC DISEASE
BACTERIAL ENDOCARDITIS
TRICHINOSIS

SUB UNGCAL HEMORRHAGE


- SPLINTER HEMMORAGE
- LINEAR RED
- EMBOLIC
BEUSS LINE
ACUTE SEVERE ILLNESS
- TRANSVERSE DEPRESSION

7. WRIST ROM DORSIFLEX, PALMARFLEXION, ULNAR AND RADIAL DEVIATION
CONDITIONS
A.CHRONIC ARTHRITIS
PAINFUL AND FUSIFORM ENLARGMENT
B. NON-SUPPURATIVE TENOSYNOVITIS
PAINFUL SWELLING ANATOMIC SNUFFBOX
- (SAUSAGE LIKE SWELLING)
- TENDON SHEATHS (EXTERNAL POLLICIS BREVIS AND ABDUCTOR POLLICIS LONGUS)
- +CREPITUS
- DUE TRAUMA AND INFLAMMATION (GOUT AND GONOCCOCAL)
C. CARPAL TUNNER SYNDROME

- COMPRESSION, NEUROPATHY MEDIAN NERVE

- (+) TPN (TINGLING, PAIN, NUMBNESS)

- FLEXOR CARPI RADIALIS AND FLEXOR CARPI ULNARIS

- PHYSICAL SIGN


ATROPHY


HYPERSTHESIAS


PROGESSIVE WEAKNESS AND AWKWARDNESS

NOTE:

43
1.
2.

TINELS SIGN TINGLING PALMARIS LONGUS


PHALEN SIGN HYPEREXTENSION BOTH HANDS FOR 3 MINS (PRAYER SIGN)


8. FOREARM
- MOST SUSCEPTIBLE FRACTURE
- VOLAR MASS FORM BY FLEXORS
- SMITHS FRACTURE


9. ELBOW
- SWELLING MORE COMMON EXTENSOR SURFACE
- RHEUMATOID NODULE FOUND OLECRANON BURSAE AND ULNAR REGION
NOTE:
HUMERO-ULNAR = EXTENSION AND FLEXION
HUMERO-RADIAL = PRONATION AND SUPINATION



CUBITUS VALGUS = OUTWARD (ANGLE LESS THAN 170)

CUBITUS VARUS = INWARD



OLECRANON BURSITIS = STUDENT MINERS
ND

ARTHRITIS ELBOW = TENNIS 2 TENDONITIS

10. SHOULDER

WINGED SCAPULA
- SPRENGELS SCAPULA
- PARALYSIS LONG THORACIC NERVE
- CONGENITAL
- SOMETIMES ASSOCIATED WITH SHORT WEBBLED NECK

11. SPINE

NODDING AND LIFTING = ATLANTO-OCCIPITAL
FLEXION AND EXTENSION = C3 AND C7
LATERAL BENDING = MIDCERVICAL
ROTATION = ATLANTO-AXIAL (C1-C2)

CONDITIONS:
PANCOAST
- SUPERIOR PULMO SULCUS SYNDROME
- TUMOR PULMO APEX, UPPER MEDIASTINUM AND
SUPERIOR THORACIC APERTURE
+NECK PAIN
PARESIS OR ATROPHY OF ARM MUSCLE MAY OCCUR
HORNER
UNI MIOSIS, PTOSIS OF EYELIDS
-SWEAT
TUMOR LUNG APEX OR NECK
CERVICAL SPONDYLOSIS
DEGENERATION OF NUCLEUS PULPOSUS
CERVICAL OSTEOARTHRITIS
WHISPLASH
RUPTURE OF LIGAMENTUM NUCHAE
SUDDEN FORCEFUL HYPEREXTENSION OF NECK WITH
HYPERFLEX RECOIL

POST TRAUMA
HYPERREFLEXION OF NECK C5
PARTIAL DISLOCATION FROM HYPEREXT UNABLE TO NOD
FRACTURE ATLAS SEVER OCCIPITAL HEADACHE



THORACOLUMBAR CONDITIONS
1. WHIPLASH CERVICAL GRADUAL NUMBNESS AND TINGLING SENSATION THAT GOES DOWN TO THE HAND
2. KYPHOSIS FORWARD DEFORMITY OF THORACIC (HUNCHBACK)
3. SMOOTH CURVE
4. ANGULAR CURVE - +GIBBUS
5. LORDOSIS POSTERIOR CONCAVITY OF LUMBAR SPINE, DEEP FURROW, POT BELLY
6. SCOLIOSIS FEMALE WALK SEXY, CONGENITAL, PARALYSIS BACK OR ABDOMINAL MUSCLE

LOWER EXT

1. HIP JOINT AND THIGH

TEST FOR HIP
LESAQUES TEST
STRAIGHT LEG RAISING
SCIATIC NERVE
TEST THE RANGE OF HIP FLEXION
PATRICS
LATERAL ROTATION OF THE HIP
GRAELENS
PASSIVE HYPEREXTENSION
ACTIVE HYPEREXTENSION
ONLY TEST IN PRONE

2. KNEE
CONDITIONS
GENU VARUM
BOW LEG
LEGS DEVIATE TOWARD MIDLINE
1. RICKETS
2. OSTEITIS DEORMANS
3. COWBOY AND JOCKEY
GENU VALGUM
KNOCKED KNEE
LATERAL DEVIATION OUTWARD
GENU RECURVATUM
KNEES FIXED IN HYPEREXTENSION WITH LITTLE ABILITY
TO FLEX
ABSENSE PATELLA

1. Which of the following disease manifest a big hand, big bones and muscles?
a. Gigantism b. Acromegaly c. Polydactyl d. Marfan syndrome
2. What is the action of axiohumeral muscle group?
a. Internal rotation of shoulder
b. External rotation of shoulder
c. Shrug shoulder at the back
d. All of the above
3. What is the action of axioscapular muscles? (same choices)
4. This disease is cause of adenoma in pituitary gland related to hypothyroidism?
a. Cretinism b. Ehler danlos syndrome c. mongolism
5. The following are changes seen in carpal tunnel syndrome?
a. Thenar atrophy b. Hypothenar atrophy c. Ape hand d. All of the above
6. Which of the following has a smooth curve of spine?
a. Cancer b. Infectious spondylitis c. Ankylosis spondylitis d. Potts disease

44

45

7. Which of the following has an angular curve of spine?


a. Ostetitis deformans b. Ankylosis spondylitis c. Senile osteoporosis d. Infectious spondylitis
8. Manifestation of Achilles tendon damage . Positive simmond test . Body weight is applied to base of the foot because of
plantar flexion . Because of hyper extension and dorsiflexion . A and b only
9. This is associated with eunochoidism
a. Eggshell nail b. Spoon nails c. Long narrow nails d. Square and round nails
10. Sub ungal haemorrhage is the cause of the following disease except?
a. Cardiac disease b. SBE c. Trichinosis d. None of the above
11. This line is associated with acute severe illnesses Beaus Line This is associated with renal disease something?haha!
forgot exact na sakit a. Red half moon b. Blue half moon c. Onychauxis d. None of the above
12. Test for lateral rotation
a. Graenlen b. Stretching of legs upward c. Patrick d. Owen
Test for hip flexion (same choices)
13. Type 3 pitting edema is?
a. 2mm b. 4mm c. 6mm d. 8mm
14. This is a congenital disease associated with damage to long thoracic nerve? Winged scapula
15. Location of tumor in pancoast syndrome
a. Apex b. Mediastinum c. Base d. A and b e. All of the above
16. Responsible for movement of neck from left to right
a. C3-c7 b. Atlanto occipital c. Axial d. Midcervical vertebrae
17. Scoliosis can be?
a. Paralysis of back muscle b. Congenital c. 1 leg is paralyzed d. A and b e. All of the above
18. Genu recurvatum
a. Fixed hyperextension b. Fixed flexed c. Congenital d. A and c
19. Morrant baker cyst is associated with? Popliteal artery syndrome
20. Genu Valgum?
a. Lateral deviation of leg b. Bow leg c. Cause of rickets d. . ?
21. Genu Varus
a. Can be cause of occupational b. Pagets disease c. Can be cause by rickets d. All of the above
22. Associated with Marfan Syndrome
a. Long slender nails b. Arachnodactyl c. Claw hand d. A and b e. All of the above
23. Cause of claw hand a. Brachial plexus injury b. Syringomyelia c. Muscle dystrophy d. All of the above
24. Nodes found in DIP is associated with?
a. Osteoarthirits b. Systemic rheumatic arthritis c. Gout arthritis d. Infectious arthritis
25. Combing of hair is associated with?
a. Internal rotation and abduction b. External rotation and abduction c. Internal rotation and adduction d. External
rotation and adduction
26. Movement of the right arm?


a. Counter clockwise on pronation and clockwise on supination b. Counterclockwise on supination and clockwise on
pronation c. Both are correct d. None of the above
27. What should you assess when looking at the spine sideward?
a. Height of shoulder
b. Curvature
c. Location of ASIS
d. All of the above

28. What should you assess if you are looking at the back of patient?
a. Height of shoulder
b. Dimples?
c. ASIS
d. All of the above

30. If a person cannot raise greater than 10 degrees in elbow joint?
a. Dislocation
b. Fracture
c. Damage of supraspinatus tendon
d. All of the above

31. Heloma Durum is?
a. Hard corn
b. Callus
c. Soft corn
d. Stiffened toe

32. If there will be a fracture in humerus, what should you check?
a. Brachial artery
b. Radial artery
c. Brachioradial artery
d. ?
Medicine Extremities
1. Dorsiflexion Soleus Tear
2. Housemaids anterior knee cyst
3. To test range of hip flexion, nerve irritation and lumbar herniation straight leg test
4. Test done to detect excessive lumbar convexity / lordosis Hugh Owen Thomas sign
5. Yellow discoloration in palms imparted to the skin by carotene carotenemia
6. Inspection of vertebral column viewed laterally AOTA, cervical, thoracic , lumbar curve
7. True about lordosis except: - posterior convexity of lumbar sign
Posterior concavity of lumbar sign
Deep furrow between paraspinous muscle
Pot belly
8. Test for hip dislocation trendelenbergs sign
9. What is the joint that allows free movement like knee and shoulder joint fibrous
Synovial
Cartilaginous
10. SITS muscle of rotator cuff muscle except: - supraspinatus
Infraspinatus
Teres minor
Subscapularis Muscle
None
11. ROM of ELBOW extension , pronation flexion, supination

46

47
12. Drop arm definition (bates) - if patient cannot hold arm fully abducted at shoulder level, possible rotator cuff
tear.
13. This can be seen in osteoarthritis - heberdens

14. Bouchards - rheumatoid arthritis
Asymmetric deformitiesof the hands and wrists
All
15. Range of motion for the wrist: flexion, extension, abduction, adduction
16. Hypertrophy of nailplates caused by chronic fungal infection onychauxis
17. Long narrow nail plates except: cretinism
18. Pronated wrist drop radial nerve palsy
19. Rotation of neck rotation at c1 and c2
20. Difference in shoulder length is due to : scoliosis
Sprengels shoulder





A and b





NONE
21. View person from spine is for : cervical convexity
Thoracic convexity



Lumbar convexity



None
22. Matching type: hugh owen
23. Stretch sciatic muscle straight leg raising
24. Lateral rotation grip patricks test
25. Passive hyperextension graelens test
26. Eversion talipes valgus
27. Dorsiflexion, defect on neural arch talipes calcaneus
28. Genu varum occur in rickettes
Pagets

Both
29. Length of lower extremities from ASIS to medial malleolus
30. Thumping of posterior spine volar surface of wrist
31. Spoon nail iron deficiency
32. Test for acromioclavicular joint cross over test
33. Bowleg genu varum
34. Bitten nails common to personality disorder
35. Knocked knee genu valgum
36. Onychauxis chronic fungal infection
37. Subungal hemorrhage splinter hemorrhage linear red hemorrhage in nailbed
38. Square and round nail plates cretinism

Unequal Leg Length can be seen - Scoliosis, Hip Fracture, Poliomyelitis except: Kyphosis

48


EXTREMITIES
1. Lumbar concavity:
false regarding lordosis(dapat lumbar concavity)
2. Soleus tear:
extreme dorsiflexion
3. Flexion and extension occur primarily between the skull and C1, the atlas; rotation at C1-C2; both
4. Sits muscles:
supraspinatus,infraspinatus,teres minor and subscapularis
5. In osteoarthritis, Heberdens nodes at DIP joints, Bouchards nodes at the PIP joint.
In rheumatoid arthritis, symmetric deformity in the PIP,MCP and wrist joints with ulnar deviation.
6. Wrist movement:
flex,extend, abduct, adduct
7. Wrist drop:
radial nerve
8. Unequal shoulder heights seen in scoliosis;:
Sprengels deformity of the scapula(from the attachment of an extra bone or band between the upper scapula
and C7); in winging of the scapula (from loss of innervations of the serratus anterior muscle by the long
thoracic nerve); and in contralateral weakness of the trapezius
9. Anterior knee cyst:
prepatellar(housemaids knee) and infrapatellar(clergymans knee)
10. Genu varum:
legs deviate towards the midline
11. Long narrow nails:
all except cretinism(included: Marfans, eunochoidism, hypopituitarism)
12. Palmar erythema:
except hemolytic anemia(included: pregnancy, portal HPN, vulvar heart dse)
13. Motion of elbow:
supination, pronation, flexion, extension
14. Thomas sign:
test for lumbar lordosis
15. Lesaques test
16. Synovial:
freely movable(knee, shoulder)
17. Scoliosis, kyphosis, hip fracture
18. You may wish to percuss the spine for tenderness by thumping, but not too roughly with your fist
19. Patricks test
20. Dorsiflexion:
either ankle motion or talipes calcaneus
21. Housemaids:
anterior knee cyst
22. To test range of hip flexion, nerve irritation, and lumbar herniation:
straight leg test
23. Test done to detect excessive lumbar convexity/lordosis:
Hugh owen Thomas sign
24. Yellow discoloration in palms imparted to the skin by carotene:
carotenemia
25. Inspection of vertebral column viewed laterally:

49
26.

27.
28.
29.
30.
31.
32.
33.
34.
35.
36.
37.
38.
39.
40.
41.
42.
43.
44.
45.
46.
47.
48.
49.
50.

AOTA, cervical, thoracic, lumbar curve


True about lordosis except:
posterior convexity of lumbar sign, post concavity of lumbar sign, deep furrow between paraspinous muscle,
pot belly
Test for hip dislocation:
trendelenbergs sign
What is the joint that allows free movt like knee and shoulder joint:
fibrous, synovial, cartilaginous
SITS muscle of the rotator cuff muscle:
supraspinatus, infraspinatus, teres minor, none
ROM of elbow:
extension, pronation, flexion, supination
Drop arm definition(BATES)
This can be seen in osteoarthritis:
heberdens, bouchards, asymmetric deformities of the hands and wrists, all
Range of motion of the wrist:
flexion, extension, abduction, adduction
Hypertrophy of the nail plates caused by fungal infection:
onychauxis
Pronated wrist drop:
radial nerve palsy
Rotation of the neck:
rotation at c1 and c2
Difference in shoulder length is due to:
scoliosis, sprengels shoulder, a and b, none
Long narrow nail plates except: Cretinism
View person for spine is for: cervical convexity, thoracic convexity, lumbar convexity, none
Matching type: hugh owen
Stretch sciatic nerve:
straight leg raising
Lateral roattion grip:
Patricks test
Passive hyperextension:
graenlens test
Eversion:
talipes valgus
Dorsiflexion, defect on neural arch:
talipes calcaneus
Genu varum occurs in:
rickettes, pagets, both
Length of the lower extremities:
from ASIS to medial malleolus
Thumping of posterior spine:
volar surface of wrist
Spoon nail:
iron deficiency
Test for acromioclavicular joint:
cross over test

50
51. Bowleg:
genu varum
52. Bitten nails common to:
personality disorder
53. Knocked knee:
genu valgum
54. Onychauxis:
chronic fungal infection
55. Subungal hemorrhage:
splinter hemorrhage linear red hemorrhage
56. Square and round nailplates:
cretinism
57. Dorsiflexion:
either ankle motion or talipes calcaneus
58. Test done to detect excessive lumbar convexity/lordosis:
hugh owen Thomas sign
59. True about lordosis except:
posterior convexity of lumbar sign
60. Test for hip dislocation:
trendelenbergs sign
61. Joint that allows free movement like knee and shoulder joint:
synovial joint
62. ROM of elbow:
extension, pronation, flexion, supnation
63. Rotaion of neck:
rotation at c1 and c2
64. Difference in shoulder length:
scloiosis, sprengels shoulder
65. Lateral rotation:
patricks test
66. Passive hyperextension:
graenlens test
67. Eversion:
talipes valgus
68. Stretch sciatic nerve:
straight leg test











51
1.

One of the following regarding examination of spine is incorrect


a. From the side inspect the spinal profile
b. From behind inspect lateral curves
c. Percuss the spine with a neurological hammer
d. Inspect and palpate paravertebral muscles
2. Conditions that impair range of motion of fingers and wrist are:
a. Arthritis
b. Tenosynovitis
c. Fibrous in the palmar fascia
d. AOTA
e. A & B
3. All of the following are associated with Marfans syndrome Except:
a. Thumb sign
b. Hyperextensible joints
c. Hypertrophic osteoarthropathy
d. Elongated long bones
4. The following statements about carpal tunnel syndrome are correct Except:
a. There is atrophy of the radial half of thenar eminence
b. Pain especially at night is experience
c. Weakness and loss of finer movements is not common on this disease
d. Trauma from excessive flexion of the wrist is common cause
5. Which of the following statements is associated with clawhand:
a. Claw is formed by hyperextension of the interphalangeal joint
b. The condition maybe secondary to Rheumatoid arthritis
c. Brachial plexus injury is one of the possible cause
d. AOTA
6. Ruptured Achilles tendon is manifested by:
a. pain at the heel
b. inability to plantar flex the foot
c. inability to dorsiflex the foot
d. shortening of the tendon
7. Pain on elevation of the arm 60-120 degrees is due to:
a. Chronic tendinitis
b. Partial rupture of of supraspinatous tendon
c. Dislocation
d. Complete fracture
8. One of the following is a cause of kyphosis in post menopausal
a. Osteoporosis
b. Faulty posture
c. Osteitis deformans
d. Ankylosing spondylitis
9. Gibbus deformity maybe secondary to:
a. Pagets dse
b. Hypercalcemia
c. Metastatic carcinoma
d. Cervical spondylosis
10. Internal rotation of the shoulder joint is tested by:
a. Raising both arms to a vertical position

52

11.

12.

13.

14.

15.

b. Putting both hands behind the small ___ of the back


c. Putting both hands behind the neck
d. NOTA
st
Among the tests of the hip joint the most gentle test that should be done 1 is:
a. Anvil test
b. Active hyperextension
c. Graenlens test
d. Rotation of the thigh
Deviation of the legs toward the midline , leads to condition called:
a. Genu valgum
b. Bowlegs
c. Knock knee
d. Genu recurvatum
Popliteal artery entrapment syndrome maybe secondary to:
a. Prepatellar bursitis
b. Popliteal abscess
c. Clergymans knee
d. NOTA
The landmarks of the shoulder joint are:
a. Coracoid process, medial epicondyle and greater tuberosity
b. Acromion, coracoid process, greater tuberosity of humerus
c. Greater tuberosity, lateral epicondyle, clavicle
d. NOTA
Excessive transverse growth of the nail plate causing the lateral edge to lacerate
a. Onychauxis
b. Onychocryptosis
c. Rams horn nail
d. onychogryphosis


53
1.
2.
3.
4.
5.
6.
7.
8.
9.
10.
11.
12.
13.
14.
15.
A.
B.
C.
D.
E.
F.
G.
H.
I.
J.
K.
L.
M.
N.
O.
P.
Q.


Rheumatoid Arthritis
Acromegaly
Mongolism
Palmar Erythema
Phalens Sign/ Tinels Sign
Trendelenberg test/ Anvil test
Lesaques sign
Hugh Owen Thomas Sign
Patricks test
Winged Scapula
Talipes Equinus
Talipes Varus
Eggshell nails
Carotenoderma
Thumb sign/ Wrist Sign

Pituitary tumor
Straight Leg Raising
PIPJ
Lateral Rotation of hip
Toe wear of shoes
Short fat hand
Yellow vegetables
Hip dislocation
Flexure contracture of hip joint
Liver Cirrhosis
Arachnodactyly
Sprengels deformity Median Nerve
Median nerve
Lateral wear of shoes
Active hyperextension
Little finger is curved
Vitamin A deficiency

Answer Key
C
A
P
J
M
H
B
I
D
L
E
N
Q
G
K

54




1.
2.
3.
4.
5.
6.
7.
8.
9.
10.
11.
12.
13.
14.
15.
A.
B.
C.
D.
E.
F.
G.
H.
I.
J.
K.
L.
M.
N.
O.















Rheumatoid Arrthritis/ Haygarths Node


Acromicria
Mongolism
Palmar Erythema
Tinels Sign
Simmonds test
Lesaques sign
Hugh Owen Thomas test
Graenlens test
Azure Half moon
Whiplash Injusry
Nodding and lifting the head
Fracture of humeral back
Winged scapula
Olecranon Bursitis

Little finger is curved
PIPJ
Passive hyperextension shoulder pain small hand
Flexion contracture, hip joint
Straight leg raising
Ruptured ligamentum nuchae
Median nerve
Achilles tendon
Hepatolenticular regeneration
Cirrhosis
Short thick hands
Atlanto occipital joint
Paralysis long thoracic nerve
Miners elbow
Atlanto axial joint

Answer Key
B
E
A
L
I
J
G
F
C
K
H
N
D
O
P

55

EXTREMITIES
1. Popliteal artery entrapment syndrome maybe due to:
A.Housemaids cyst
B. Infrapatellar cyst
C. Popliteal abcess
D. Large Bakers cyst
2. Splinter hemorrhage in nailbeds is suggestive of:
A..SubAcute bacterial endocarditis B. Anemia C. Hepatolenticular degeneration D. Iron deficiency
3. Congenital small patella

A.Genu Varum B. Genu Recurvatum
C. Genu Valgus D. Degenerative osteoarthritis
4. The thumb is held in extension by its inability to flex

A.Ape hand
B.Claw hand
C.Wrist drop
D. Benediction Hand
5. In performing the ROM of the spine the examiner should assist the elderly patient by

A.Holding one hand of the patient B.Asking him to be careful C.Asking him to hold Hand Rails
D.Stabilizing the pelvis by holding the pelvis of the patient while patient performs ROM
6. Lesion of the Ulnar Nerve leads to:

A.Wrist drop
B. Claw hand
C. Hypothenar atrophy D. Palmar Erythema
7. As the examiner positions himself behind the patient, he does the ff:

A.Inspect for lateral curves
B. Locate the dimples of venus
C. Take note of the height of shoulders and iliac crest

D.Inspect cervical, thoracic and lumbar curves
8. When looking for rheumatic nodules the most likely area to look is:

A.Medial Epicondyle
B. Extensor surface near olecranon process C.Knee Joint
D. Lateral Epicondyle
9. The paravertebral muscles are evaluated by:

A.Inspection for fasiculations
B. Ascultation for crepitus C. Palpate for tenderness and spasm
D.All are correct
10. Motions of the forearm:

A.Pronation & Supination B. Flexion & Extension C. Both
D. Neither
11. The popular theory as to the cause of clubbing of the finger is/are:

A.Floating nail B.-20 degress angulation of the nail
C.hypozia
D. All
12. Haygarths Node except

A.Distal Interphalangeal joint B. Rheumatoid arthritis C.Prodromal Interphalengeal joint D. Metacarphophalengeal gout
13. A casue of kyphosis in young males due to painful disease of the spine is

A.Pagets Disease B. Osteoporosis C. Faulty posture D.Ankylosis spondylitis
14. Legs are outwardly deviated:

A.Genu recurvatum
B. Genu varus C. Genu valgum D. Osteoarthritis Knee
15. Nodding and lifting the head involves:

A.Midcervical Vertebra B. C3-C7 C. Atlantooccipital joint D. Atlantoaxial joint
16. Conditions that impair range of motion of the hands and wrists because of scarring

A.Arthritis
B.Dupuytrens Contracture
C. Tendenitis
D. ALL
17. Carpal Tunnel Syndrome:
st
nd
rd
th

A.Numbers if the Small & Ring fingers B. Numbness of the 1 , 2 ,3 & 4 fingers C. Wrist drop D.ALL
18. Ruptures ligamentum nuchae

A.C5 fracture
B. Cervical Spondylosis C. C1 fracture D. whiplash cervical injury
19. A line drawn across the iliac spine approximates the
nd
th
th
st

A.2 lumbar vertebra
B.4 lumbar vertebra
C.7 thoracic vertebra
D.1 lumbar Vertebra
20. Absence of plantar flexion

A.Tibial Fracture
B.Ruptured tendon od Achilles
C. Ruptured Bakers cyst D. Soleus tear











56

CHEST, LUNGS, BOOBS



Medicine I
nd
Midterms 2 Semester (February, 2013)

1. Acute onset but chronic progressive.
-
SMOKING
2. Which is likely the mechanism of cough among patients with heart failure?
-
DECREASED LUNG COMPLIANCE
3. Which is likely the mechanism of cough among patients with heart failure?
a. Pressure in the airway
b. Decreased lung compliance
c. Inflammation of the airway
d. Thermal change
4. 50 y.o. patient male with > 5 months cough. He cough when he lays down wearing tight clothes. It is productive with
watery whitish sputum.
a. Congestive heart failure
b. Gastroesophageal regurgitation
c. Bronchiectasis
d. Intestinal fibrosis
5. 45 year old male has cough for 3 months now and claims to clear his throat every morning
a. Smoking
b. Post Nasal Drip
8. A 62 years old female was found unconscious and at the ER she was noted with very fast and deep breathing with
fruity breath. Her pattern of breathing is:
a. Biots respiration
b. Tachypnea
c. Kassmauls respiration
d. Hyperpnea
9. 17 yo female consulted at the ER because of high grade fever w/o other symptoms. She is observed w/ abnormal
respiration. Her abnormal respiration expected to be:
a. Tachypnea
b. Platypnea
c. Hyperpnea
d. Kussmauls
10. An IV drug user presents to ER due to hemoptysis and DOB but denies any fever and cough. What is the most
common cause of hemoptysis?
a. PTB
b. Lung Abscess
c. Pneumonia
d. Vasculitis
12. Kussmauls respiration is heard among patient with:
a. Meningitis
b. Ascites
c. CVD
d. Renal failure
13. 38 y/o female, rushed to ER because of severe retrosternal pain, relieved by nitroglycerine; pain occurs when cold
substance ingested.
a. Angina
b. Trachitis
c. Esophagitis
d. Esophageal spasm
14. Acute onset of Difficulty of Breathing precipitated by valsalva maneuver
a. Pulmonary edema
b. Rupture of aortic aneurism

15.
16.
17.
18.

19.
20.
21.
22.
24.

26.

27.

29.
31.

33.

34.

57
c. Pneumothorax
d. Pulmonary emboli
Most common cause of hemoptysis
-
Bronchiectasis
Patient with heart failure
-
Orthopnea
Dry cough and unilateral effusion
-
Trepopnea
Pneumococoniosis is due to exposure to:
a. Silica
b. Coal
c. Asbestos
d. Silver
Bagasse
-
Sugar cane
Primary malignancy
-
Silica
Psittacosis
-
Parrot
Lagging of one side of the chest is best identified by:
-
Palpation
Where is the landmark for the posterior rib?
a. C7
b. T1
c. Superior scapular line
d. Inferior scapular line
Referenced used in identifying posterior ribs
a. C7
b. T1
c. Superior scapular line
d. Inferior scapular line
Tactile fremitus is usually decreased with:
a. Atelactasis
b. Emphysema
c. Bronchitis
d. Pneumonia
Stridor
-
Epiglottis
Normal breath sound heard in intrascapular area
a. Vesicular
b. Bronchovesicular
c. Bronchial
d. Tracheal
Sounds of secreations produce on bronchi?
a. Stridor
b. Rhonchi
c. Rales
d. Wheezes
Bilateral hyperresonance
a. Tension pneumothorax
b. Pneumomediastinum
c. Chronic bronchitis
d. Emphysema


35. Findings on the exam of the anterior chest would mostly involved which of the following:
a. Upper lobe
b. Middle lobe
c. Lower lobe
d. A & B
36. Shorter I:E ratio
a. Metabolic acidosis
b. Bronchitis
c. Pleurisy
d. Fever
37. Describe the type of breathing for patient with heart failure:
a. Cheyne-stokes
b. Biot
c. Kusmail
d. Apneustic
38. Fremitus is decreased
a. Pneumonia
b. Obesity
c. Pleural effusion
d. Atelectasis
40. Early inspiratory crackles is heard at:
a. Interstitial lung disease
b. Pneumonia
c. CHF
d. Chronic bronchitis
41. Breast extends from?
nd
th
-
2 to 6 ICS
42. Most frequently palpable lymph node in Breast Ca:
-
Central Lymph Node
43. Mastitis
-
Lactation
44. Dividing into quadrants is toL
a. Clinical finding
b. Staging
46. Seen in acute onset and chronic progressive DOB:
a. Bronchiectasis
b. Asthma
c. Pneumonia
d. Anemia
47. Orange peel in breast cancer:
a. Protrusion of mass
b. Lymphatic abscess
c. Venous abscess
d. Increased blood flow


Q: Mediastinal mass cough, described as signs of respiratory distress, except:

Q: Diaphragmatic excursion except:
a. Atelectasis
b. Pneumonia
c. Pneumothorax
d. Pleural effusion

Q: A 35 y/o consulted for fear of breast cancer. Her sister died of breast cancer at 23 y/o. What is the risk?

58

59
a.
b.
c.
d.

>40
2.1-4
1.1-2
None


Q: Suggestive of barky cough

Q: Brassy cough
A: Emphysema

Q: Brassy
A: Trachea

Q: Barky
A: Glottis

Q: Pink puffer
A: Emphysema

Q: Mastitis
A: Lactation

Q: Man cant keep up with friend at same age
A: Grade 2

Q: Occupational asthma common among workers exposed to which of the following dust:
a. Asbestos
b. Coal
c. Cotton dust
d. Silicon

Q: Pneumocosis
A: Coal

Q: Heart Failure
A: Orthopnea

Q: Apex above clavicle
A: 2-4cm

Q: COPD
A: 10x

Q: Lung cancer in women
A: 13x


Q: Basilan
A: Paragoniasis

Q: Rusty color of sputum
A: S. pneumonia

Q: Pursed lip
A: Emphysema



Q: Crescendo=decrescendo with apnea
A: Cheyne strokes

Q: Afferent limb
A: IX

Q: Decreased or low fremitus
A: Thick chest wall

Q: Soft and low
A: Vesicular

Q: Early inspiratory crackles
A: Chronic bronchitis

Q: Most common palpable lymph node of breast cancer
A: Central Node

Q: High (4x) breast density
A: Highest to relative risk for breast cancer

Q: Modifiable risk for breast cancer
A: Obesity

Q: Chronic cough
A: 2 months

Q: Cough complication
A: Pneumothorax

Q: IE ration is decreased in fever
A: Chronic bronchitis

Q: Chest pain associated with chest tenderness pathology of
A: Chest wall

Q: Pursed lip breathing is manifested by patients with:
a. Bronchiectasis
b. Asthma
c. Emphysema
d. Pneumonia

Q: Pink puffer also known as
a. Asthma
b. Emphysema
c. Chronic bronchitis

Q: Below are hereditary diseases except
a. HPN
b. Leprosy
c. Heart Disease
d. DM

60

61


BREAST
1. Intraductal Papilloma
> SPONTANEOUS UNILATERAL BLODDY DISHARGE FROM ONE/TWO WARANTS FURTHER EVALUATION
2. 1.1 2.0
> OBESITY, LATE AGE ETC.
3. Surface of Areola
> SMALL, ROUNDED ELEVATION FORMED BY SEBACCEOUS GLANDS , SWEAT GLANDS & ACCESSORY AREOLAR
GLANDS
4. Tail of Spence
> EXTENDS ACROSS THE ANTERIOR AXILLARY FOLD
5. Female Breast
ND
TH
> CLAVICLE, 2 RIB DOWN TO THE 6 RIB AND FROM THE STERNUM ACROSS TO THE MIDAXILLARY LINE


Recall Clinical Medicine Chest and Lungs (Dec. 17, 2012)

1. During cough, the speed of airflow in the airway is as fast as the speed of:

a. Light



c. Airplane

b. Sound



d. Lightning

2. What is etiology of cough in pleural effusion?

a. Inflammation


c. Decrease pulmonary compliance

b. Mechanical irritation

d. Chemical/thermal

3. 18 y.o. female with acute onset of cough associated with runny nose, sore eyes, and chest discomfort. What condition?

a. Pneumonia


c. Mycoplasm pneumonia

b. Acute bronchitis


d. Tracheobronchitis

4. Chronic cough, sputum purulent, often copious and foul-smelling

a. Bronchiectasis

c. Obstructive pneumonia

b. Chronic bronchitis


d. PTB

5. Cause of brassy cough

a. Glottis



c. Bronchiole

b. Trachea



d. Lung parenchyma

6. Patient with which of the following lung problem would present with sudden onset of difficulty of breathing preceded
by pleuritic chest pain?

a. Pulmonary edema


c. Pulmonary emboli

b. Pneumothorax


d. Pulmonary hemorrhage

7. Sudden onset of difficulty of breathing precipitated by exposure to grass pollen

a. Asthma



c. Pneumonia

b. COPD


d. Hypersensitive Pneumonitis

8. Alpha I antitrypsin increase risk:

a. Asthma



c. Chronic bronchitis

b. Cystic fibrosis


d. Emphysema

9. 49 y.o. DB, he has DOB for 3 years. Lately, he cant keep up with his friends of the same age. What is the grade or
American Thoracic Dyspnea Scale?

a. Grade 1



c. Grade 3

b. Grade 2



d. Grade 4

62

10. Considered as the most common cause of hemoptysis?



a. Pneumonia


c. Bronchiectasis (not sure)

b. PTB



d. Lung CA

11. Lung parenchyma, common cause of hemoptysis?

a. Bronchiectasis


c. Bronchogenic CA (not sure)

b. Good pasteurs syndrome

d. Cystic fibrosis

12. Squeezing retrosternal pain felt after ingestion of cold liquid and relieved by nitroglycerin is felt in?

a. Heart



c. Chest Wall

b. GIT



d.

13. 40 y.o. female with rheumatoid arthritis of more than 15 years now and taking methotrexate. Started to have cough
and difficulty of breathing 2 years ago. What respiratory disease is the most likely the cause?

a. Non-cardiogenic pulmonary edema
c. Pulmonary vasculitis

b. Pulmonary emboli


d. Interstitial infiltrative disease

14. Which of the following occupational inorganic substances will cause pneumoconiosis?

a. Bagasse



c. Coal

b. Asbestos



d. Silica

15. Exposure to which of the following dusts increases the risk of lung cancer?

a. Bagasse



c. Coal

b. Tin oxide



d. Silica

16. Cigarette smoking increases risk of COPD mortality by:

a. 2.3x



c. 10x

b. 4x



d. 13x

17. Smoking increases lung cancer mortality in men by:

a. 4x




c. 20x

b. 6x



d. 23x

18. Alcoholics have higher risk of getting pneumonia and they are more prone to develop:

a. Aspiration pneumonia

c. Pneumonia due to tularemia

b. Psittacosis



d. Pneumocystic pneumonia

19.To decrease tension pneumothorax where will you insert the big bore needle:
nd
th

a. 2 ICS MCL (Mid Clavicular Line)
c. 6 ICS AAL (Anterior Axillary Line)
th
th

b. 4 ICS MAL (Mid Axillary Line) d. 8 ICS MAL

20. Which of the following vertebrae has the most prominent spinous process

a. C6



c. T1

b. C7



d.

21. Needle insertion of thoracocentesis
nd
th

a. 2 ICS



c. 6 ICS
th
th

b. 4 ICS


d. 8 ICS

22. The apex of the lungs is extended above the inner 1/3 of the clavicle by as much as:

a. 1-3cm



c. 3-5cm

b. 2-4cm


d. 4-6cm

23. Diaphragmatic excursion from resting to full inspiration is at:

a. 1-3cm



c. 5-6cm

63


b. 3-5cm



d. 6-7cm

24. Bifurcation of trachea at:
nd

a. Manubrium


c. 2 ICS
rd

b. Sternal Angle


d. 3 Rib

25. Pursed lip breathing

a. Emphysema


c. Asthma

b.




d.

26. Blue bloater

a. Chronic bronchitis


c. Bronchiectasis

b. Emphysema


d.

27. Ribs angle is usually more horizontal compared to normal individual if patient have which of the following problem:

a. Pneumonia


c. Pneumothorax

b. COPD


d. Mild Asthma

28. Normal angle of the rib:

a. 35 degrees



c. 45 degrees

b. 40 degrees



d. 50 degrees

29. Narrowing of ICS may cause shift to the chest to the one side:

a. Massive effusion


c. Disc atelectasis

b. Severe pneumonia


d. Obstruction of main bronchus


30. Unilateral widening and even bulging of ICS is suggestive of:

a. Massive effusion


c. Large lung mass

b. Severe pneumonia


d. Pneumothorax

31. IE ratio in emphysematous patient:

a. 1:2



c. 1:4

b. 1:3 (not sure)


d. 1:5

32. Which among the following abnormal respiratory rhythm is seen in the lesion of the PONS?

a. Cheyne stroke


c. Kussmaul

b. Biots



d. Apneustic

33. Diabetic Ketoacidosis:

a. Cheyne stroke


c. Kussmaul

b. Biots



d. Apneustic

34. Type of respiration associated with morphine overdose:

a. Cheyne stroke


c. Kussmaul

b. Biots



d. Apneustic

35. Clubbing of fingers are seen in this pathologic, except:

a. Bronchiectasis


c. Mesothelioma (not sure)

b. COPD


d. Pulmonary fibrosis

36.
37. Which of the following breath sounds have intermediate pitch and loudness?

a. Tracheal



c. Broncho-vesicular

b. Bronchial



d. Vesicular



38. Which of the following breath sounds has loud and high pitched sound?

a. Tracheal



c. Broncho-vesicular

b. Bronchial



d. Vesicular

39.
40. Mid inspiratory & expiratory crackles usually heard in patients with:

a. Bronchiectasis

41. Normally breast tissue extend from the sternum to:

a. AAL



c. PAL

b. MAL



d. Between AAL & MAL

42. Rounded elevations seen in the areola is caused by the following except:

a. Sebacceous glands


c. Accessory glands

b. Sweat glands


d. Fat globule

43. Fibroadenoma of the breast is a common cause of breast mass among female aging:

a. 15-20 y.o.



c. 50-65 y.o.

b. 25-50 y.o.



d. >65 y.o.

44. 40 y.o. nullipara, married, no family history of breast CA, what is the risk factor?

a. >4



c. 1.1-2.0 (not sure)

b. 1-4



d.

45. Which of the following can decrease the risk of breast CA?

a. Use of contraceptive

c. Breast feeding

b. Late pregnancy



d. Post menopausal

46. MRI of the breast is recommended for the following except:

a. Older woman


c. High risk group

b. Newly diagnosed breast CA

d. Women with dense breast

47. Galactorrhea may be seen in patient with

a. Herpes zoster


c. Ovarian CA

b. Pituitary adenoma


d.

48. Benign condition of breast may cause bloody breast discharge

a. Adenofibrosarcoma


c. Fibroadenoma

b. Intraductal papilloma

d. Neurosarcoma

49. Breast malignancy is most likely to have if breast tumor is about:

a. 2cm



c. 4cm

b. 3cm



d. 5cm (not sure)

50. Breast malignancy is highly considered if the patient have

a. Smooth border


c. Palpable mass

b. Wall delineated


d. Immobile






64

65

CARDIOLOGY

Areas of Auscultation
nd
a. 2 ICS LSB
nd
b. 2 ICS RSB
c. Erbs Point
th
d. 4 ICS LSB
th
e. 5 ICS LMCL
nd

36. Pulmonary Stenosis 2 ICS LSB


th
37. Mitral Stenosis 5 ICS LMCL
nd
38. Opening Snap 2 ICS LSB
th
39. Mitral valve prolapse 5 ICS LMCL
nd
40. PDA 2 ICS LSB
nd
41. Aortic stenosis 2 ICS RSB
th
42. S3 & S4 5 ICS LMCL
th
43. Mitral regurgitation 5 ICS LMCL
nd
44. Splitting of S2 2 ICS LSB
th
45. Tricuspid regurgitation 4 ICS LSB







Clinical Medicine: CVS exam
1 to 5 choices:
a. concentric

d. RVH
b. eccentric

e. LVH
c. LV dilatation

1. Not palpable in PE
2. Forceful apical beat but not displaced
3. Markedly displaced at the left axillary line, hardly palpable
th
th
4. Displaced to axillary line 5 or 6 ICS anterior axillary line
5. Palpable parasternally but retract in systole

6 to 20 choices:
nd
th
a. 2 Left ICS

d. 4 ICS PS
nd
th
b. 2 Right ICS
e. 5 ICS MCL
c. Erbs point

D
6. Pulmonic stenosis
C
7. VSD
D
8. ASD
A
9. PDA
10.
11. Aortic valve regurgitation
E
12. Mitral valve regurgitation
D
13. Mitral prolapse
B
14. Aortic stenosis
E
15. Mitral stenosis
E
16. Loud S1
A
17. Opening snap


A
18. Loud P2
D
19. Hypertrophic cardiomyopathy
E
20. LV hypertrophy

Multiple choice ito na super haba, eto yung summary
21. Mitral stenosis diastolic rumbling
22. Aortic stenosis transmitted to the carotid
23. Aortic regurgitation Diastolic rumbling murmur
24. VSD pansystolic

25 to 30 choices:
a. Caravallos sign
d. Gallavardins
b. Kussmauls


e. Austin-Flint
c. (+) Hepatojugular

f. Machinery-like

D
25. Aortic stenosis
A
26. Tricuspid regurgitation
F
27. PDA
E
28. Chronic aortic regurgitation
C
29. CHF
B
30. Constrictive pericarditis

31 to 35 choices:
a. Loud P2

d. Opening snap
b. Loud A2

e. Ejection Click
c. Non-ejection sound

D
31. Mitral stenosis
C
32. Mitral valve prolapse
33. Constrictive pericarditis
A
34. Pulmonary hypertension
B
35. Chronic uncontrolled hypertension

36 to 40 choices:
a. Paradoxical
b. Wide
c. Loud
d. Negative hepatojugular reflex

36. Mitral stenosis
37. Hypertensive urgency
38. Pulmonary hypertension
A
39. Severe aortic stenosis
40. Pulmonary hypertension










66


41 to 45 choices:
a. Pulsus parvus et tardus
b. Corrigan pulse
c. Pulsus bisferiens
d. Hepatojugular
e. Neck vein distention at 60

D
41. Normal PE finding
E
42. Pulmonary edema
A
43. Aortic stenosis
C
44. Combined aortic regurgitation and aortic stenosis
45. Chronic aortic regurgitation

46 to 50 choices:
a. Ejection systolic murmur best heard at
b. Persistent splitting
c. Diastolic murmur
d. Rumbling
e. Blowing

46.
A
47. Aortic stenosis
D
48. Mitral stenosis
49.Aortic regurgitation
50. Pulmonic stenosis


CVS

A.Concentric
B. RVH
C. Kussmaul Sign
D. +Hepatojugular Reflex
E. LV Dilatation




















67

68

1. 52 year old female known to be diabetic and hypertensive patient admitted because of severe dyspnea (Orthopnea),
cardiomegaly and bilateral basal crackles. Neck vein is visible and become more distended upon application of pressure in
the liver

2. 27 year old male complained of fever, dyspnea and chest pain aggravated by breathing. Apical beat is markedly
displaced to midaxillary region with distant heart sound. The patient is diagnosed with viral cardiomyopathy.

3. 42 year old male who was not aware that he was hypertensive, when seen in OPD his BP was 180/110 on sitting
position. Apex beat at 5th ICS LAAL with apical beat to be sustained and foreceful.

4. 36 year old female school teacher with history of frequent sore throat was admitted because of pedal edema, esay
fatigability and DOE. +Parasternal retraction during systole and an apical diastolic rumbling murmur

5. 65 years old male known CAD patient status post bypass graft surgery 10 years ago is now suffering from fatigue,
dyspnea, orthopnea, edema and pulmonary edema. Apical beat is hardly palpable at 6th ICS. Axillary region faintly audible
heart sounds and adynamic precordium
*****

A. Loud S1
B. Soft S1
C. Loud A2
D. Loud P2

1. Chronic regurgitation
2. Hypertensive Urgency with BP 220/120
3. Rheumatic Mitral Stenosis
4. Pulmonary edema in a patient with CHF
5. Chronic mitral regurgitation
*****

A. Physiologic Splitting S2
B. Persistent Splitting S2
C. Paradoxical Spilitting S2
D. Wide Splitting S2

1. ASD
2. Pulmonic Stenosis
3. Aortic Stenosis
4. Complete RBBB
5. LBBB
****













69

A. Right mitral stenosis


B. Right aortic with mitral regurgitation
C. Congenital bicuspid aortic stenosis
D. Congenital pulmonic stenosis
E. Right aortic regurgitation

1. 36 year old male admitted because or bilateral pedal edema and orthopnea. P.E revealed a hyperdynamic apical pulse
seen adn palpable at 6th ICS LMA region and a diastolic blowing murmur at the ERBS point transmitted to the apex

2. A 42 year old GRO was admitted because of easy fatigability pedal edema orthopnea and PND which started around 6
months PTC becoming progressive. P.E revealed diastolic thrill and murmur with Loud S1 and opening snap

3. 42 year old male admitted because of chest pain and episode of syncope. P.E revealed +ejection systolic murmur at 2nd
ICS RSB transmitted to the neck

4. 30 year old female presents a persistent spilitting of S2 with expiratory splitting of S2 at the 2nd ICS LCB ejection systolic
murmur at the same time

5. 42 year old male known RHD patient was admitted because of severe dyspnea, jaundice, ascites, hepatomegaly, P.E
revealed apical beat markedly displaced at the 6th ICS left midaxillary region with apical lift, apical diastolic blowing
murmur transmitted from the axilla and posterior scapula

Significant Clinical Findings
1. Important physical examination finding in CHF (+) hepatojugular reflux
2. Systolic ejection murmur of chronic severe aortic regurgitation is accompanied by which sound Austin-Flint
murmur
3. Mid-diastolic murmur heard over the pulmmonic area is ASD is accompanied by which sound Persistent
splitting of S2
4. Increased loudness of holosystolic murmur of tricuspid regurgitation upon inspiration Caravallos sign
5. Heard in aortic stenosis Paradoxical splitting of S2

Cardiomyopathies
th
6. Apex beat is displaced at the 6 ICS LAAL, (+) para sterna heave, (+) lift Eccentric
7. 40 year old female, with apical beat that retracts during systole Right Ventricle Hypertrophy
8. Hardly palpable Left ventricle dilatation
th
9. 40 year old female with chronic hypertension complained of chest pain, PE revealed apex beat at the 5 ICS LMCL
with a (+) apical lift Concentric
10. A 52 year old, Male, chronic hypertensive and DM2 complained of chest pain and SOB (DOB?), PE revealed (+)
th
murmur of AR and MR, apex beat is displaced at 5 ICS left anteroaxillary line with (+) lift. Eccentric

Abnormal Sounds
11. Pulmonary Hypertension loud P2
12. Mitral stenosis Loud S1
13. Atrial Fibrillation Soft S1
14. BP 220/120 loud S1
15. Pulmonic stenosis Paradoxical splitting of S2
16. Sinus Tachycardia loud S1
17. Chronic aortic regurgitation Loud A2
18. LBBB Paradoxical splitting of S2
19. Severe aortic stenosis Loud A1
20. Normal PE findings splitting of S2 upon inspiration



70

Abnormal Pulses
21. Chronic aortic regurgitation and aortic Stenosis- Pulsus Bisfierens
22. Aortic regurgitation Corrigans pulse
23. Constructive pericarditis Pulsus Paradoxus
24. Severe aortic Stenosis Pulsus parvus et tardus
25. CHF pulsus alterans

Cases
26. Diastolic blowing murmur aortic regurgitation
th
27. 4 ICS parasternal line Tricuspid regugitation
28. Ejection systolic murmur transmitted to carotid aortic stenosis
29. Ejection systolic murmur loud on expiration pulmonic stenosis
30. Holosystolic murmur displaced to left axilla, diminished S1 Mitral regurgitation



1. Important physical examination finding in CHF
+ Hepatojugular reflux
2. Mid diastolic murmur heard over the pulmonic area in ASD is accompanied by what sound?
Persistent splitting of S2
3. Loudness of holosystolic murmur of tricuspid regurgitation upon inspiration
Caravallos Sign
th
4. Apex beat is displaced at the 6 ICS LAAL, faint heart sound, (+) parasternal heave, (+) lift.
Eccentric Hypertrophy
5. 40 year old female, with apical beat that retracts during systole
RVH
th
6. SOB, apex at 6 ICS left midaxillary line..faint heart sound, alcoholic and elicit drug use
Eccentric
th
7. 40 year old with chronic hypertension complained of chest pain. PE revealed apex beat @ 5 ICS LMCL with a (+)
apical lift.
Concentric
8. A 52 year old Male, chronic hypertension & DM2 complained of chest pain and SOB. PE revealed (+) murmur of
th
AR & MR. Apex beat is displaced @ 5 ICS left anteroaxillary line with (+) lift.
Eccentric
9. Pulmonary hypertension
Loud P2
10. Atrial fibrillation
Soft S1
11. BP 220/120
Loud S1
12. RBBB persistent

13. Sinus Tachycardia
Loud S1
14. Chronic Aortic Regurgitation
Soft S1
15. LBBB
Paradoxical splitting of S2
16. Severe Aortic Stenosis
Paradoxical splitting of S2

71
17. Normal PE findings
Split S2 on inspiration
18. Chronic aortic regurgitation and aortic stenosis
Pulsus bisfiriens
19. Severe aortic stenosis

20. CHF
Pulsus Alterans


21. A 27 year old female, worried about ECG finding of RBBB. Auscultation revealed presence of ejection systolic
murmur which became louder during forceful expiration.
Pulmonic stenosis
22. 65 year old patient with DM, HPN, has a holosystolic murmur displaced to the left axilla, has a diminished S1
Mitral regurgitation
23. Machinery-like murmur
PDA
24. Greater BP in the upper extremities compared to the lower extremities
Coarctation of Aorta
25. Characteristics peripheral pulse in CHF
Pulsus Alterans
26. DM hypertensive 64 year old experienced pain in right calf muscle on walking for 20 mins and relived by rest
Intermittent Claudication
27. PDA
nd
2 Left ICS
28. Aortic Stenosis
nd
2 Right ICS
29. S3 & S4
th
5 ICS LMCL
30. Mitral Regurgitation

31. Fixed splitting of S2
ASD
32. Holosystolic murmur at the left parasternal area are transmitted to the right sternal border
VSD
33. Sail Sound
Ebsteins anomaly
34. BP elevated in the upper extremities markedly lower in the lower extremities
Coarctation of Aorta
35. Machinery-like murmur
PDA
36. Pulmonary Stenosis
nd
2 Left ICS






Clinical Medicine : CARDIOVASCULAR SYSTEM EXAM
For 1 to 5, choices are:
a. concentric

d. RVH
b. eccentric

e. LVH
c. LV dilataion

1. Not palpable in PE C
2. Forceful apical beat but not displaced. A
3. Markedly displaced at the left axillary line, hardly palpable. C
4. Displaced to axillary line 5th or 6th ICS anterior axillary. B
5. Palpable parasternally but retract in systole. D

For 6 to 20, choices are:
a. 2nd LEFT ICS
d. 4th ICS PS
b. 2nd RIGHT ICS
e. 5th ICS MCL
c. Erb's point

6. Pulmonic Stenosis A
7. VSD C
8.
9.
10.
11. Aortic valve regurgitation A or D
12. Mitral valve regurgitation E
13. Mitral Prolapse E
14. Aortic stenosis B
15. Mitral stenosis E
16. Loud S1 E
17. Opening snap A
18. Loud P2 A
19. Hypertrophic Cardiomyopathy D
20. LV Hypertrophy E

multiple choice ito na super haba, eto yung summary:
21. Mitral stenosis - diastolic rumbling
22. Aortic stenosis - transmitted to the carotid
23. Aortic Regurgitation - diastolic rumbling murmur
24. VSD - pansystolic

For 25 to 30, choices are:
a. Caravallo's

d. Gallavardin's
b. Kussmaul's

e. Austin-Flint
c. + hepatojugular
f. Machinery-like

25. aortic stenosis D
26. tricuspid regurgitation A
27. PDA F
28. chronic aortic regurgitation E

72


29. CHF C
30. constrictive pericarditis B

For 31 to 35, choices are:
a. loud P2

d. opening snap
b. loud A2

e. ejection click
c. non-ejection sound
31. Mitral stenosis D
32. Mitral valve prolapse C
33. Constrictive pericarditis E
34. Pulmonary Hypertension A
35. Chronic uncontrolled hypertension B

For 36 to 40, choices are:
a. Paradoxical
b. Wide
c. Loud
d. Negative hepatojugular reflex
36. Mitral stenosis C (Loud S2)
37. Hypertensive urgency
38. Pulmonary hypertension C (Loud S2)
39. Severe aortic stenosis A (Paradoxical splitting of S2)
40. Pulmonary hypertension C (Loud S2)

For 41 to 45, choices are :
a. Pulsus parvus et tardus

d. negative hepatojugular
b. Corrigan's pulse


e. neck vein distention at 60 degrees
c. pulsus bisferiens
41. normal PE finding D
42. Pulmonary edema E
43. Aortic stenosis A
44. Combined aortic regurgitation and aortic stenosis C
45. Chronic aortic regurgitation B

For 46 to 50, choices are:
a. ejection systolic murmur best heard at
b. persistent splitting
c. diastolic murmur
d. rumbling
e. blowing
46. Mitral regurgitation D
47. Aortic stenosis A
48. Mitral stenosis C
49. Aortic stenosis E
50. Pulmonic stenosis B


73

74
1.

2.

3.

4.
5.
6.
7.
8.
9.
10.

11.
12.
13.
14.
15.
16.
17.
18.
19.
20.

The anterior portion of the heart


a. LV
b. RV
c. Aorta
d. Pulmonary Artery

The diameter of the apical beat
a. 1.5cm
b. 2.5cm
c. 2.0cm
d. .5cm

Patient with COPD has palpable pulse at the epigastric and subxiphoid region
a. Abdominal aorta
b. RVH
c. LVD
d. Biventral hypertrophy

Mitral stenosis



a. loud s1
CAR





b. soft s1
Atrial fibrillation



c. loud s2
MSD




d. ejection







a. deep inspiration






b. forceful expiration
MVP




c. leaning forward
d .LL decubitous

Einsteins Anomaly



a. Caravallas
Chronic Tricuspid Regurgitation

b. Galavardins
Aortic Stenosis



c. Snail sound
Chronic Severe Aortic Regurgitatin

d. Paradox pulse

nd
Opening snap at rheumatic mitral stenosis
a. 2 LICS
nd
Pulmonic regurgitation


b. 2 RICS
VSD





c. Parasternal
MVP




d. Erbs
Congenital Pulmonic Stenosis


e. Apex

Position of the apical beat during physical exam
a. upright, leaning forward
b. supine
c. LLD




75
21. Normal location of the apical beat
a. Strong and forceful
th
b. Always at 5 ICP, L MCL
c. Gentle tap
d. Strong during inspiration

22. Concentric LV

23. Ischemic Dilated Cardiomyopathy
24. Chronic Severe Aortic Stenosis
25. ASD



26. Normal

a. Persistent S2
b. Paradox S2
c. Physiologic S2
d.



1.
2.
3.
4.
5.
6.
7.
8.
9.
10.
11.
12.
13.
14.
15.
16.
17.
18.



Ventricular Septal Defect


> HEARD @ APEX
Very loud S1
> BEST HEARD ON APEX
Opening Snap
> BEST HEARD ON PULMONIC AREA
Apical mid-diastolic rumbling murmur
> BEST HEARD ON APEX
Chronic Aortic Regurgitation
> ERBS POINT
Acute Severe Mitral Regurgitation w/ s3 Gallop
RD
> 3 LICS
RightBundle Branch Block
> PERSISTENT / WIDE S2 SPLITTING
LeftBundle Branch Block
> PARADOXICAL S2 SPLITTING
Non-Systolic Click
> (MVP) ERBS POINT
Less Prolapse: SQUAT ; More Prolapse: STAND
Ejection Systolic Murmur
(HEARD LOUDEST @ BASE)
Atrial Septal defect
ND
> 2 LICS
Concentric L-ventricular Hypertrophy
th
> 5 ICS LAAL
Restrictive Hypertrophy Cardiomyopathy
> STANDING SQUATTING POSISTION
Kussmaul Sign
> CAUSE BY INABILITY OF THE (R) HEART TO ACCOMODATE INCREASE VENOUSE RETURN
Loud/accentuated S1
> HYPERTENSION
Physiologic Splitting
> 2-3 (L) ICS
Paradoxical Splitting of S2
> (L) BUNDLE BRANCHING BLOCK

76


ABDOMEN

36-40: Draw and label the 4 quadrants of the abdomen and its landmarks:
40-50: Draw and label the nine regions of the abdomen and its landmarks:


Abdomen Finals Samplex
1. A palpable left flank mass is probably the left kidney if:
a. Youre palpating fingers can probe deep to the medial and lateral borders (splenomegaly to)
b. Preservation of normal tympany in LUQ
c. A notch is palpated on medial border (splenomegaly din)
d. Edge of mass extend beyond the midline
2. An extremely tight sphincter tone on rectal examination is due to:
a. Neuropathy (seen in lax sphincter to)
c. Cerebrovascular disease
b. Spinal cord lesion

d. Anxiety
3. A 35 y/o male complains of sudden severe epigastric pain radiating to the back, duration noted after a heavy
meal. If this is acute abdominal pain, the following statement is/are correct
a. Always mandate surgical intervention
c. Can be manage medially
b. Duration of pain is less than 2 days
d. Consider if the pain is severe
4. A 21 y/o male with acute leukemia have a palpable splenic notch. The traubes percussion will be:
a. Hyperresonant


c. Resonant
b. Dull




d. Tympanic
5. A 60 y/o diabetic had stroke 1 month ago with right sided hemiparesis and dysarthria..for sensation of food wont
go down and just stay on the mouth and repeatedly attempts to swallow. This type of dysphagia is:
a. Oropharyngeal


c. Esophageal dysphagia
b. Esophageal web
6. In patient complaining of burning sensation that begins inferiorly and radiates up to the entire retrosternal area
to the neck should avoid the following except:
a. NSAIDs

c. Meperidine
e. Citrus food
b. Coffee

d. Beta blockers

______________________________________________________________________________________

Abnormal contour xymphoid to symphisis pubis
Portal HPN:Except - SMJ nodule
Puddle sign flicking sound becomes louder, as the stet goes farther
Hypoactive bowel Ruber test
Abdominal paradox chest in, abdomen expand upon inspiration
Ovarian cyst and ascites hypokalemia, furosemide, abdominal distention
Difference of voluntary form involuntary rigidity Inspire with wide open mouth
Carnettes sign Abdominal vs Intramural tendencies
Normal liver span 4-8 cm Midsternal, 6-12 RMCL
Normal size of aorta 3 cm
To palpate liver:except Castell technique (splenomegaly)
th

Kidney punch 12 rib, costovertebral angle


Methods to assess Appx: except Boas sign


Sims position Left knee flex, side of the table
Rectal exam in women mass, tenderness, nodules, lateral wall, uterus
Acute prostatitis Boddy and tender
Grade 3 3-4 cm protrusion
Murphys sign Inspiratory arrest
Using Ulnar surface of the pt hand at midline fluid wave
_____________________________________________________________________________________

Alcoholic portal hpn massive ascites except:
a. shifting dullness


c. everted umbilicus
b. fluid wave



d. Typhanitic

Nixon lateral decubitus

To confirm secondary hpn
a. Epigastric


c. R/L iliac

e. All
b. R/L upper q


d. A and B

Obese pt
a. light palpate



c. Reinforced palpate
b. deep palpate



d. Ballotment




ABDOMEN
1. Abdominal contour Xyphoid to symphysis pubis
2. Portal Hypertension Except: SMJ nodule
3. Puddle Sign Flicking sound becomes louder, as the stet goes farther
4. Hypoactive bowel Hypokalemia, Furosemide, Abdominal distention
5. Ovarian cyst and ascites Ruler Test
6. Differentiate voluntary to involuntary rigidity Inspire with wide open mouth
7. Garnetts Sign Abdominal versus intramural tenderness
8. Normal liver span 4-8 cm Midsternal; 6-12 RMCL
9. Normal size of the aorta 3cm
10. To palpate liver Except: Castells technique (for splenomegaly)
th
11. Kidney punch 12 rib, costovertebral angle
12. Method to assess AP (Appendicitis) Except: Boas Sign
13. Sims Position Left knee flexed side of the table
14. Rectal exam in women Mass, tenderness, nodules, lateral wall, uterus
15. Acute prostatitis Boggy and tender
16. Grade 3 3-4cm protrusion
17. Murphys sign Inspiratory arrest
18. Using ulnar surface of patient hand in the midline Fluid wave

77

78


ABDOMEN
1. In doing fluid wave, the ulnar surface of the patients hand is pressed into the midline of the abdomen to
a. Prevent movement of intestines
b. Direct the movement of the wave to the opposite side
c. Block the movement of the mesenteric fat
d. Clearly visualize the fluid wave

2. A 60 yo CHF for which he was maintained on Furosemide developed hypokalemia. He has abdominal distention
without abdominal pain. The auscultatory findings will be
a. Succusion splash
b. Hyperactive bowel sounds
c. Hypoactive bowel sounds
d. Normal bowel sounds

3. A smoker with emphysema was noted to have palpable liver 3 finger breaths below the right subcostal margin.
The expected liver span is
a. 4-8 cm midsternal
b. 6-12 MCL
c. 8-14cm anterior axillary line
d. All

4. All but one are expected findings in a 46 yo male alcoholic with portal hypertension
(All are expected findings in a 46 yo male alcoholic with portal hypertension EXCEPT)
a. Globular abdomen with everted umbilicus
b. Sister Mary Joseph nodules
c. Spider angioma
d. Obliterated Traubes space
e. (+) Castellss sign percussion

5. A 19 yo male have severe attacks of bronchial asthma was noted to have abdominal paradox. The abdominal
respiratory motion is
a. Abdomen contract with expiration
b. Rocking motion of chest and abdomen
c. Abdomen expands while chest is pulled inward
d. All

6. To differentiate ascites from large ovarian cyst, you must do
a. Ballotment palpation
b. Deep palpation
c. Shifting dullness
d. Fluid wave
e. Ruler Test




79
7.

8.

9.

10.

11.

12.

13.

Carnetts sign will differentiate (please double check na lang po yung correct answer, thanks!)
a. Intraabdominal mass from intramural mass
b. Rebound tenderness from direct tenderness
c. Intraabdominal tenderness from abdominal tenderness
d. Subcutaneous crepitus from cutaneous hyperthesia
A 45 yo female complaining of RUQ pain was diagnosed to have acute cholecystitis. A positive Murphys sign is
a. Tenderness on RUQ
b. Inspiratory arrest
c. (+) fist tenderness on percussion of right subcostal margin
d. Exaggerated pain on gentle lifting of a fold of skin on RUQ

In a 22 yo male with RLQ pain suspected to have acute appendicitis, the ff are positive abdominal signs, EXCEPT
a. Markles
b. Blumbergs
c. Aarons
d. Boas
e. Obturator

Positive puddles sign is
a. Distinct tap on your palpating hand
b. The sound becomes louder while the stethoscope moves away from the flicking spot
c. Dullness shift to the dependent side while tympanitic shift to the top
d. A visible movement as a tap was done on the other side of the puddle

In doing the kidney punch, a direct percussion with the fist should be applied to
a. Subcostal margin
b. Flank
th
c. 12 rib and vertebral angle
d. ASIS

A 66 yo male with enlarged prostate Grade III on rectal examination, have this amount of protrusion
a. 1-2 cm
b. >2-3 cm
c. >3-4 cm
d. >4 cm

A 30 yo male complaining of dysuria and fever with pyuria on urinalysis. The expected rectal examination findings
if this is acute bacterial prostatitis is/are
a. Sulcus is obliterated
b. >1 cm protrusion
c. Nodular
d. Boggy and tender
e. All




80
14. The ff statements is/are true with regards to significance of rectal examination in female EXCEPT
a. Cervix may be palpable through anterior wallt
b. Uterus is never palpable on rectal examination
c. Tenderness of peritoneal inflammation can be appreciated
d. Nodularity of peritoneal metastasis can be felt

15. Sequence of abdominal examination
ANS. Inspection, Auscultation, Palpation, Percussion, Special Examination
16. Reference used in determining abdominal contour is an imaginary line drawn from
a. Breast to umbilicus
b. Clavicle to the symphysis pubis
c. Rib margin to umbilicus
d. Xiphoid or rib margin to the pubis

17. To differentiate between abdominal rigidity amd voluntary muscle guarding, examiner should do this while doing
palpation
a. Distract the patient by conversation
b. Ask the patient to breathe with mouth wide open and feel relaxation of abdomen during expiration
c. Ask patient to raise his head from supine position
d. All
e. A and B only

18. Technique to palpate a large organ is freely movable as mass obscured by ascites
a. Capture technique
b. Deep palpation
c. Reinforced palpation
d. Ballotment

19. Sims position of patient in rectal examination
a. Bent over the examining table
b. Lateral position lying on the left side with right hip and knee flexed with buttocks close to edge of table
c. Knee-chest position
d. Lithotomy position

20. An enlarged prostate gland on rectal examination has the following findings EXCEPT
a. Firm, rubbery consistency
b. Obliteration of sulcus
c. >1cm protrusion into the rectum
d. Lateral lobes are palpable


Abdomen
A.MATCHING TYPE
D1.Ovarian cyst vs Ascites
A2.Obesity vs. Ascites
B3.Intramural vs. Intraabdominal mass
C4.Peritonitis vs. Organomegaly
E5.----------------
A. Inverted umbilicus
B. Lift head while in supine position
C. Carnettes sign
D. Ruler test
E. Traubes space
B.MULTIPLE CHOICE
6.Visceral pain has the following characteristics
A. More intense and more localized
B. Patient move about in an effort to relieve the discomfort
C. Aggrevated by moving or coughing
D. Felt in areas remote to diseased organ
7.Acute abdominal pain,except
A. Approximately <24hours
B. Sudden in onset and generalized
C. mandate surgical procedure
D. All of the above
8.Apley rule
A. The nearer from the navel the pain,the more likely it will be organic in origin
B. The farther from the navel the pain,the least likely it will be organic in origin
C. The nearer from the navel the pain,the least likely it will be organic in origin
D. The farther from the navel the pain, the more likely it will be organic in origin


9.Post myocardial infarction, (+) gag reflex
A. Pyrosis
B. Globus pharyngeus
C. Oropharyngeal dysphagia
D. Esophageal dysphagia
10.It is the sensation of a lump or tightness in the throat unrelated
A. Pyrosis
B. Globus pharyngeus
C. Oropharyngeal dysphagia
D. Esophageal dysphagia
11.Causes of heartburn,except
A. Meperidine
B. NSAIDs
C. Coffee
D. Beta blockers
E. Citrus food
12.A positive Guiac test
A. Hematemesis
B. Melena
C. Hematochezia
D. Occult bleeding


81


13.Type of diarrhea that has inhibition of sodium absorption
A.Secretory
B. Osmotic
C. Acute
D. Chronic
14.Etiologic agents in acute diarrhea,except
A. Vibrio
B. Salmonella
C. Hepatitis
D.None of the above

15.Rome 2 classification of Chronic constipation
A. No bowel movement
B.Straining
C. No flatus
D. Abnormal bowel sounds
16.A delay between attempting to initiate urination and actual flow of urine
A. Overflow incontinence
B. Stress incontinence
C.Hesitancy incontinence
D. Reflux incontinence
17.Occurs in coughing,laughing,exercise,lifting heavy objects
A. Overflow incontinence
B.Stress incontinence
C. Hesitancy incontinence
D. Reflux incontinence
18.Color of spun urine is red
A.Pigmenturia
B. Hematuria
C. Both
D. Neither
19.By indirectly stimulating the chemoreceptor trigger zone,vomiting may be initiated by followingdrugs
A. Anti-arrhythmias
B. Macrolides
C. Oral hypoglycemic agents
D. Opiods
20.Causes of hypoactive bowel sounds
A. Diarrhea
B. Early intestinal obstruction
C. Hypokalemia
D. Laxatives
21.A poitive cullen sign
A. Massive non-traumatic ecchymoses
B. Acute hemorrhagic pancreatitis
C. Noted on the skin of the lower abdomen and flanks
D. Blue-red,blue-purple or green-brown in color
22.A strangulated hernia
A. Richters hernia
B. Usually is almost always incarcerated
C. Resembles pain of chronic peptic ulcer
D. All of the above



82

83

23.Sounds produced by large collection of air and fluid from stomach or intestine
A. Borborygmi
B. Succussion splash
C. Peritoneal friction rub
D. Fibrillary hum
24.Markels sign is
A. Jar tenderness
B. Superior to rebound tenderness as a localizing sign of peritoneal irritation
C. Used for ambulatory patients with rigid abdominall muscles
D. All of the above
25.In a patient with abdominal aortic aneurysm
A. There is periumbilical mass
B. There is upper abdominal mass
C. With expensile pulsations
D. All of the above
26.Normal abdominal movement with respect to respiration

27.Direction of blood flow in the abdomen


28.A palpable splenic tail is
A. Dull in percussion note
B. Suggests splenic infection
C. Tympanitic in traubes space
D. Flat in percussion note
29.Until how many cm is the liver edge palpable at the RUQ
A. 1cm
B. 2cm
C. 3cm
D. 4cm
30.A normal prostate consists of the following, except
A. Firm and rubbery
B. About 2.5cm in length
C. Measure 4 x 3 x 2cm
rd
D. 3 or median lobe is composed of glandular tissue which is palpable
31.An example of referred pain
A. Pulmanary tuberculosis
B. Myocardial infarction
C. Peritonitis
D. Pancreatitis
32.The liver is palpable of about how many cm during inspiration below the right costal margin on the right midclavicular
line?
A. 3-4 cm
B. 4-8cm
C. 6-12cm
D. 4-6cm
33.Bright red stool usually is associated with
A. Lower GI bleeding
B. Upper GI bleeding
C. Massive upper and lower GI bleeding
D. None of the above




34.Liver tenderness is assed by
A. Deep palpation
B. Ballottement
C. Reinforced palpation
D. All of the above
35.Ballottement is used to determine a large organ, which is freely movable, or a mass obscured by
A. Peritonitis
B. Appendicitis
C. Ascites
D. Pancreatitis
C.DRAW AND LABEL
40-45. Draw and label the 4 quadrants of the abdomen and its landmarks
46-50. Draw and label the nine regions of the abdomen and its landmarks



Clinical Medicine Abdomen
1. Nocturia awakening the patient more than
a. Once (ans) b. Twice c. 3 d. 4
2. If the liver is not palpable. To check for liver tenderness you should elicit?
A. Cva tenderness B. Percuss the traube's space. C. First in the percussion. D. Murphy's sign
3. Signs ascites upon inspection of abdomen
a. globular abdomen b. bulging flanks c. everted umbilicus d. a& b e all of the above
4. Abdominal hernia wherein blood supply is interrupted.
Answer: strangulated
5. True about acute abdominal pain.
Answer: pain less than 24 hours
6. Palpation of normal prostate gland.
Answer: firm, rubbery, smooth and non tender
7. Upon rectal examination the finger of the examiner can reach up to
Answer: 6-10cm
8. All but one important sa patient na may abdominal pain something
Answer: Touch me not warning
9. Manifestation of paralytic ileus.
Answer: hypo or absent bowel sound.
10. Renal bruit can be heard at.
Answer: RUQ/LUQ
11. Rulers test.
Answer: large ovarian cyst
12. Position of the patient on abdominal examination.
Answer: supine with arms in side or over the chest with pillow under the head and knee
13. Hematemesis
Answer: UGIB
14. Abdominal pain of acute peritonitis?
aggravated by movement / coughing
15. Pattern for abdomen examination
ANSWER: Inspection, Aus, palpation, percussion, special examination
16. Bowel sounds can be heard
ANSWER: RLQ (Not sure kung may ganito)
17. Best/preferred position for rectal examination of ambulatory patients
ANSWER: Bend over the examining table position
18. Present as tuft of engorged abdominal veins radiating from the umbilicus
ANSWER: CAPUT MEDUSA
19. Dysphagia within 1 second of swallowing
ANSWER: ORAPHARYNGEAL DYSPHAGIA

84



Abdomen
th
1. Kidney Punch- 12 rib costovertebral angle
2. Nodular mass (prostate) except- 3-4cm node?
3. Acute bacterial prostate- Tender and Boggy
4. Rectal tenderness- except: Palpation of cervix?
5. Sequence of abdominal examination- I, A, Pal, Per
6. Fluid Wave- Tapping 1 side of abdomen transmited to the opposite side
7. Pain RLQ (Appendicitis) except: Boas Sign
8. Puddle Sign- + flicking is repeated while stetoscope is more farther away
9. Abdominal contour margin- Xiphoid to Symphysis Pubis
10. Sims Position-lateral postion lying on his left side with right hip and knee somewhat flexed
11. Normal liver span-6-12cm MCL
12. Prostate enlargement 3-4cm rectal protrusion- Grade 3
13. Abdominal Paradox-..
14. Carnett sign- differentiates abdominal tenderness due to inflamed abdominal wall
15. Ascities vs Ovarian Cyst- Rulers Test





Questions 1-10 matching type
1.fluid wave- NO VISIBLE WAVE! hindi umaalon ang abdomen accdg to macy
2. Aaron's sign- precordial pain
3. Murphy's sign- respiratory arrest
4. Boa's Sign
5. Carnett-patient raises his head
6. Castell
7. Psoas
8. Puddle
9. Rovsing

11. Which among the ff applies to Apley Rule?
A. Pt may give a touch me not sign
B. Organic dse usually keep the pt eyes open
C. The farther from the umbilicus, the more organic
D. Pt with organic cause is generally not hungry

85


12. 77 y/o male, smoker, developed dysphagia that progressed to include liquids. What type of disorder?
A. Transfer
B. Oropharyngeal
C. Motility
D. Esophageal (accdg sa pinakamamahal kong scholar na si larizza, eto daw po sagot)
13. Characteristics of Irritable Bowel syndrome
14. How much urine volume is excreted by a person who is oliguric? <500mL
15. Dysphagia with psychological history? Globus Pharyngeus
16. 17,male, excreting blood per orem, differentiate hematemesis from hemoptysis?
A. Color of blood
B. Look for associated symptoms
C. Do NGT
D. Correlate clinical findings
Ans. Accdg again kay Lare, B sinagot nya PERO, di sya sure kasi mukhang ok din ung D.
P.S Roi, ikaw na sumagot, nakalimutan ko tanungin si Macy at Ana tungkol dito
17. Patient passing out tarry, black, foul smelling stool,. Where is the bleeding?
A. Upper GIT
B. Lower GIT
C. Obscure
D. A and B
E. All
Sabi ni Macy E daw, sabi ni Roi E din daw, sabi ko,mkinig sa higher center ^_^
18. Patient with heartburn, you should inquire if patient is taking all but one in the history taking:
A. Coffee
B. KCl tablets
C. Theophylline
D. Omeprazole

86


19. 25 year old male with dysuria what should you inquire about?
A. Urethral Discharge
B. Sexual practices
C. Renal calculi
D. A and C
E. All
20. 72 yr old with dribbling, distended bladder upon PE, What kind of incontinence?
Ans. Overflow incontinence
21. Looking for secondary cause of HPN, where to look for bruit?
Ans. Epigastric and RUQ/LUQ
22. sound produced upon auscultation, due to large air and fluid production. Can be detected by unaided ear
Ans. Succussion splash
23. Sign of acute hemorrhagic pancreatitis: Cullen's
24. Paradoxical breathing: Rocking motion (sorry nakalimutan na yung ibang choices)
25. In portal HPN, direction of flow of upper abdominal veins? Drains upward
26. Patient with history of redness in the urine. Differentiate hematuria and pigmenturia
A. Presence of blood clots
B. Foamy urine
C. Increase in intensity of color when left standing
D. Reddish brown sediments formed















87

88

ABDOMEN

1. A patient with ascites & obliterated Traubes space have a prominent abdominal superficial veins. The expected
direction of blood flow on this vessel will be
a. upward on upper abdominal veins


c. lateral at middle abdominal veins
b. downward on lower abdominal veins

d. all over

2. Normally, the abdominal respiratory motion is/are
a. rocking motion of chest & abdomen


d. all
b. abdomen expand with inspiration



e. b&c


c. abdomen contract with expiration


3. In umbilical hernias, if the contents is incarcerated & blood supply is interrupted, it is called
a. reducible




c. irreducible
b. incarcerated



d. strangulated

4. A smoker with COPD on abdominal exam has a palpable liver 3 finger breathe below the right subcostal margin is
expected to have a liver span of
a. 4-6 cm on midsternal
c. 8-13cm on anterior axillary line e. a & b only
b. 6-12 cm on MCL

d. all

5. A 21 y/o male with acute leukemia have a palpable splenic notch. The Traubes space percussion will be

a. dull


c. resonant

b. tympanitic


d. hyperresonant

6. To assess liver tenderness in a patient with a liver abscess when the liver is not palpable, you should do

a. deep palpation on RUQ
c. reinforced palpation

b. capture technique

d. fist percussion of right subcostal margin

7. In patient with gastric outlet obstruction, the sound produced by large collection of air and fluid from stomach,
detected by unaided ear is

a. venous hum

c. succussion splash

b. borborygmi

d. tinkling bowel sounds

8. During abdominal examination of patient with generalized abdominal pain, you ask patient to stand on his toes &
suddenly drop his heels on the floor. You are eliciting

a. Blumberg sign

c. Psoas sign

b. Rovsings sign

d. Markles sign

9. A hypertensive 78 y/o male is being evaluated for abdominal aortic aneurysm. If this is present, the expected findings
will be:

a. width of aorta is >3cm

b. direction of aortic pulsation is directly on the palpating fingers

c. lateral pulsation of aorta is noted

d. all

e. a & b only


10. An 80y/o male, developed anuria & complaining of hypogastric pain. Rectal examination revealed grade 4 prostate
enlargement. The expected abdominal finding/s is/are
a. palpable mass on hypogastric area


d. all
b. dullness on percussion of hypogastric area


e. a & b
c. symmetrically globular abdomen


11. Normal prostate gland on rectal examination has the following findings, except

a. firm and rubbery in consistency
c. nontender and movable

b. prominent median sulcus

d. 5cm diameter with >1cm protrusion

12. On rectal examination the examining finger can palpate a distance of

a. 2-4 cm

b. 4-6 cm
c. 6-10 cm
d. 8-12cm

13. On PE of the abdomen, auscultation is done before doing palpation because:

a. it will prevent ticklishness of the patient

b. palpation alters frequency of bowel sounds

c. more convenient for the examiner

d. it is more preferred by the patient

14. Reference used in determining the abdominal contour is an imaginary line drawn from:

a. rib margin to the umbilicus

c. xiphoid to rib margin to the symphysis pubis

b. breast to the umbilicus

d. clavicle to the symphysis pubis

15. All but one are use to overcome ticklishness of the patient on palpation of the abdomen, EXCEPT

a. ask patient to perform self-palpation

b. talk to the patient

c. place your hands over patients finger and after sometime drift slowly your fingers onto the abdomen

d. use diaphragm of stethoscope as palpating instrument

































89

90

ABDOMEN
1. PARIETAL PAIN - More intense and more precisely located - Aggravated by movement or coughing
2. ACUTE ABDOMINAL PAIN - <24 hours - Not always mandate surgery - CAN be manage medically
3. APLEYS RULE - The farther from the navel/umbilicus, the more likely it will be organic in origin (only answer) - Other
choices: - Touch me not warning - Organic disease when patient closes/opens his eyes
4. OROPHARYNGEAL DYSPHAGIA - Food wont go down - Repeatedly attempts to swallow
5. ACHALASIA - All are related to disease: - With weight loss
6. All are associated with HEARTBURN - Coffee, alcohol, NSAIDS, nitrates, theophylline - EXCEPT: - Colchicine
7. HEMATOCHEZIA - Bright red blood in rectum - Lower GI bleeding - Massive upper GI bleeding (>1L)
8. OCCULT GI BLEEDING - Chronic anemia - No change in color - (+) Guiacs test
9. BLEEDING OF OBSCURE ORIGIN - All of the above
10. PEPTIC ULCER - Upper GI bleeding or massive bleeding
11. AMOEBIC COLITIS - Small painful stool (tenesmus)
12. ALLERGY IN CRABS AND OYSTERS - Vibrio species, salmonella, hepatitis A
13. All are part of ROME 2 CRITERIA except: - Lumpy or hard stool - Sensation of incomplete evacuation - bowel action per
week
14. CONSTIPATION IN ELDERLY - Decrease food intake - Weak abdominal and pelvic muscles - Slow colonic transit
15. HESITANCY - Delay between attempting to initiate urination and actual flow of urine
16. CAUSE OF PYURIA - All of the above
17. OLIGURIA - <500ml/24hrs
18. POLYURIA - Increase osmotic load - Increase intake of fluid - ADH deficiency
19. HEMATURIA - (+) RBC sedimentation
20. OVERFLOW INCONTINENCE - A continuous dripping or dribbling incontinence
21. NIXONS TECHNIQUE - Percussion is initiated at lower level of lung resonance
22. REINFORCEMENT - Deep palpation is difficult, obesity and deep seated pathology
23. LIVER SPAN - Accurate liver size
24. SIMS POSITION - Patient lies on one side with arms behind the back and the upper thigh flexed, use in vaginal exam
MATCHING TYPE
25. Ascites and obesity - Inspection of umbilicus
26. Ascites and ovarian cyst - Ruler test
27. Intramural and intraabdominal - Patient raises his head while in supine position
28. Abdominal tenderness (muscle strain) - Carnets sign
29. Splenomegaly and hepatomegaly - Traubes space
30. PORTAL HYPERTENSION - Globular abdomen (everted) - Alcoholic; liver cirrhosis - (+) engorge abdominal veins
radiating from umbilicus
31. JAUNDICE - Can only be differentiated from carotenemia in the color of palm
32. CULLENS SIGN - Acute hemorrhagic pancreatitis
33. RESPIRATORY PARADOX - Rocking motion of chest and abdomen
34. ABDOMINAL HERNIA - Strangulated
35. SUCCESSION SPLASH - Large collection of air and fluid from stomach and intestine
36. JAR TENDERNESS



Personal note: Please, wag kayo umasa dito, maipasa niyo man ang clinmed I, babagsak parin kayo sa board exam pag to
lang aralin niyo. Kung kasing tamad kayo tulad ko, makinig ka nalang ng mabuti sa prof, 3 hours of lecture of your 24 hrs is
not so hard to give.

- N

You might also like